final

Pataasin ang iyong marka sa homework at exams ngayon gamit ang Quizwiz!

22. According to Provision 2 of the American Nurses Association (ANA) Code of Ethics, which member of the health-care team is the nurse's primary commitment? Select all that apply. 1) Patient 2) Family 3) Physician 4) Community 5) Surgeon

1,2,4 Feedback 1. This is correct. The patient is the nurse's primary commitment according to the ANA Code of Ethics. 2. This is correct. The family is the nurse's primary commitment according to the ANA Code of Ethics. 3. This is incorrect. The physician is not the nurse's primary commitment according to the ANA Code of Ethics. 4. This is correct. The community is the nurse's primary commitment according to the ANA Code of Ethics. 5. This is incorrect. The surgeon is not the nurse's primary commitment according to the ANA Code of Ethics.

The nurse is concerned that an older adult patient is at risk for developing acute renal failure. Which information in the patient's history support the nurse's concern? Select all that apply. 1) Diagnosed with hypotension 2) Recent aortic valve replacement surgery 3) Total hip replacement surgery five years ago 4) Taking medication for type 2 diabetes mellitus 5) Prescribed high doses of intravenous antibiotics

1,2,5 Feedback 1. This is correct. Older adults develop acute renal failure more frequently because of the higher incidence of serious illnesses, hypotension, major surgeries, diagnostic procedures, and treatment with nephrotoxic drugs. Decreased kidney function associated with aging also puts the older patient at risk for kidney failure. Hypotension, scheduled for aortic valve replacement surgery, and receiving high doses of intravenous antibiotics increase this patient's risk for developing acute renal failure. 2. This is correct. Older adults develop acute renal failure more frequently because of the higher incidence of serious illnesses, hypotension, major surgeries, diagnostic procedures, and treatment with nephrotoxic drugs. Decreased kidney function associated with aging also puts the older patient at risk for kidney failure. Hypotension, scheduled for aortic valve replacement surgery, and receiving high doses of intravenous antibiotics increase this patient's risk for developing acute renal failure. 3. This is incorrect. A previous history of major surgery and current treatment for type 2 diabetes mellitus are not identified risk factors for the development of acute renal failure. 4. This is incorrect. A previous history of major surgery and current treatment for type 2 diabetes mellitus are not identified risk factors for the development of acute renal failure. 5. This is correct. Older adults develop acute renal failure more frequently because of the higher incidence of serious illnesses, hypotension, major surgeries, diagnostic procedures, and treatment with nephrotoxic drugs. Decreased kidney function associated with aging also puts the older patient at risk for kidney failure. Hypotension, scheduled for aortic valve replacement surgery, and receiving high doses of intravenous antibiotics increase this patient's risk for developing acute renal failure.

A patient recently diagnosed with diabetes mellitus (DM) is hospitalized in diabetic ketoacidosis (DKA) after a religious fast. The patient tells the nurse, "I have fasted during this season every year since I became an adult. I am not going to stop now." The nurse is not knowledgeable about this particular religion. Which nursing actions would be appropriate? Select all that apply. 1) Request a consult from a diabetes educator. 2) Assess the meaning and context of fasting for this religion. 3) Tell the patient that things are different now because of the new diagnosis. 4) Ask family members of the same religion to discuss fasting with the patient. 5) Encourage the patient to seek medical care if signs of ketoacidosis occur in the future.

1,2,5 Feedback 1. This is correct. The diabetes educator should be contacted to work with the patient on strategies that might allow the fasting to occur in a safe manner. 2. This is correct. Assessing the meaning and context of fasting in the patient's religion would be educative for the nurse and an appropriate action. 3. This is incorrect. Telling the patient that life is different now does not support religious beliefs. 4. This is incorrect. Asking the family to talk to the patient might help, but the diabetes educator would be able to provide more direct and helpful information for the patient. 5. This is correct. Stressing the importance of promptly seeking care when signs of ketoacidosis occur helps to promote the patient's health and is appropriate.

The community nurse visits the home of a young child who is home from school because of sudden onset of nausea, vomiting, and lethargy. The nurse suspects acute renal failure. Which clinical manifestations support the nurse's suspicions? Select all that apply. 1) Edema 2) Wheezing 3) Hematuria 4) Postural hypotension 5) Elevated blood pressure

1,3,5 Feedback 1. This is correct. Pediatric manifestations of acute renal failure characteristically begin with a healthy child who suddenly becomes ill with nonspecific symptoms that indicate a significant illness or injury. These symptoms may include any combination of the following: nausea, vomiting, lethargy, edema, gross hematuria, oliguria, and hypertension. 2. This is incorrect. Wheezing is not a manifestation of acute renal failure. 3. This is correct. Pediatric manifestations of acute renal failure characteristically begin with a healthy child who suddenly becomes ill with nonspecific symptoms that indicate a significant illness or injury. These symptoms may include any combination of the following: nausea, vomiting, lethargy, edema, gross hematuria, oliguria, and hypertension. 4. This is incorrect. Postural hypotension is a manifestation of acute renal failure in an older person. 5. This is correct. Pediatric manifestations of acute renal failure characteristically begin with a healthy child who suddenly becomes ill with nonspecific symptoms that indicate a significant illness or injury. These symptoms may include any combination of the following: nausea, vomiting, lethargy, edema, gross hematuria, oliguria, and hypertension

Which risk factors exhibited by the patient presenting in the emergency department (ED) would place the patient at risk for metabolic acidosis? Select all that apply. 1) Pneumonia 2) Abdominal fistulas 3) Acute renal failure 4) Hypovolemic shock 5) Chronic obstructive pulmonary disease

2,3,4, Feedback 1. This is incorrect. Chronic obstructive pulmonary disease and pneumonia place the patient at risk for respiratory acidosis with the increased retention of carbon dioxide in the blood. 2. This is correct. Metabolic acidosis is rarely a primary disorder. It usually develops during the course of another disease; presence of abdominal fistulas; which can cause excess bicarbonate loss; acute renal failure; and hypovolemic shock. 3. This is correct. Metabolic acidosis is rarely a primary disorder. It usually develops during the course of another disease; presence of abdominal fistulas; which can cause excess bicarbonate loss; acute renal failure; and hypovolemic shock. 4. This is correct. Metabolic acidosis is rarely a primary disorder. It usually develops during the course of another disease; presence of abdominal fistulas; which can cause excess bicarbonate loss; acute renal failure; and hypovolemic shock. 5. This is incorrect. Chronic obstructive pulmonary disease and pneumonia place the patient at risk for respiratory acidosis with the increased retention of carbon dioxide in the blood.

2. The nurse is assessing a patient's postoperative wound and finds it has separated from the suture line with extrusion of the bowel through the opening. When documenting this finding, which term will the nurse use? 1) Wound infection 2) Wound dehiscence 3) Wound evisceration 4) Wound tunneling

Wound dehiscence is separation of the suture line without visible organs or tissues.

Which should the nurse teach the patient regarding NPO status prior to a surgical procedure? 1) Nothing by mouth for 12 hours prior to surgery 2) Nothing solid by mouth for six hours prior to surgery 3) No clear liquids by mouth for four hours prior to the surgery 4) No clear liquids by mouth for two hours prior to the surgery

and 4 The guidelines for NPO status prior to a surgical procedure is nothing solid by mouth for eight hours prior to the procedure and no clear liquids by mouth for two hours prior to the procedure. NPO status is meant to decrease the patient's risk for aspiration.

A patient diagnosed with acquired immune deficiency syndrome (AIDS) is admitted to the acute care floor. Which stance regarding the care for this patient is supported by the American Nurses Association (ANA) Code of Ethics? 1) The nurse is morally obligated to care for the patient unless the risk exceeds responsibility. 2) The nurse has the responsibility to ensure the patient gets adequate medical care. 3) The patient has the right to choose not to disclose his or her condition to staff. 4) The patient is morally bound to disclose every aspect of his or her condition to staff.

ans 1 Feedback 1 According to the ANA Code of Ethics, the nurse cannot set aside the moral obligation to care for the patient infected with human immunodeficiency virus (HIV) unless the risk exceeds the responsibility. 2 This does not reflect the stance by the ANA Code of Ethics. 3 This does not reflect the stance by the ANA Code of Ethics. 4 This does not reflect the stance by the ANA Code of Ethics.

The hospice nurse is providing care to a terminal patient who has asked about guidance and support in ending life. Which should the nurse recognize in regards to making an ethical and moral decision in this circumstance? 1) Euthanasia has legal implications along with moral and ethical ones. 2) Passive euthanasia is an easy decision to arrive at. 3) Active euthanasia is supported in the Code for Nurses. 4) Assisted suicide is illegal in all states

ans 1 Feedback 1 Determining whether an action is legal is only one aspect of deciding whether it is ethical. Legality and morality are not one and the same. The nurse must know and follow the legal statutes of the profession and boundaries within the state before making any decision. 2 Passive euthanasia involves the withdrawal of extraordinary means of life support and is never an easy decision. 3 Active euthanasia and assisted suicide are in violation of the Code for Nurses. 4 Some states and countries have laws permitting assisted suicide for clients who are severely ill, are near death, and wish to commit suicide.

An adolescent patient diagnosed with leukemia decides to stop chemotherapy treatments. The patient's parents, however, want the health-care team to continue all treatments. Which action by the nurse is appropriate when providing care to this patient and family? 1) Helping the family by providing information and allowing them to voice their concerns 2) Confronting the parents and telling them not to be "selfish" in their child's time of need 3) Calling the authorities immediately 4) Obtaining a court order to determine the patient is legally able to make his or her own decisions

ans 1 Parents have the authority to make health-care decisions for their children. Dilemmas arise when parents and children do not agree on whether or not to go forward with a recommended treatment. In most cases, the nurse and other members of the health-care team who have developed a therapeutic alliance with the child and family may be able to help the family come to a joint decision by providing additional information and opportunity to discuss their concerns with each other calmly and openly. In some cases, however, the health-care team may need to seek guidance from the agency's ethics committee.

The nurse is reviewing evidence from a quasi-experimental research study. Which level of evidence should the nurse identify for this research study? 1) Level I 2) Level II 3) Level III 4) Level IV

ans 1 A systemic review of randomized controlled studies, not a quasi-experimental research study, is identified as Level I.

The nurse is providing care for a patient admitted to the unit with respiratory failure and respiratory acidosis. Which data from the nursing history is the probable cause for the patient's current diagnoses? 1) Aspiration pneumonia 2) A recent trip to South America 3) Recent recovery from a cold virus 4) Use of ibuprofen for the control of pain

ans 1 Aspiration of a foreign body and acute pneumonia would put the patient at risk for respiratory acidosis

The nurse is caring for a patient with congestive heart failure who is admitted to the medical-surgical unit with acute hypokalemia. Which prescribed medication may have contributed to the patient's current hypokalemic state? 1) Cortisol 2) Demerol 3) Skelaxin 4) Nonsteroidal anti-inflammatory drugs (NSAIDs)

ans 1 Excess potassium loss through the kidneys is often caused by such medications as corticosteroids, potassium-wasting (loop) diuretics, amphotericin B, and large doses of some antibiotics. Cortisol is a type of corticosteroid and can cause hypokalemia.

Which ethical principle is the nurse assessing when asking who benefits from the actions of others? 1) Beneficence 2) Autonomy 3) Justice 4) Fidelity

ans 1 Feedback 1 Beneficence asks the question who benefits from the actions taken by others. 2 Autonomy examines an individual person's right to make decisions while providing acknowledgement and respect for the person's choices. 3 Justice examines who will be vulnerable from any actions taken. 4 Fidelity requires the nurse to be accountable for commitments made to others and self.

The nurse is caring for a patient admitted with hypertension and chronic renal failure who receives hemodialysis three times per week. The nurse is assessing the patient's diet and notes the use of salt substitutes. When teaching the patient to avoid salt substitute, which rationale supports this teaching point? 1) They can potentiate hyperkalemia. 2) They will cause the client to retain fluid. 3) They will increase the risk of AV fistula infection. 4) They will interact with the client's antihypertensive medications

ans 1 Feedback 1 Many salt substitutes use potassium chloride. Potassium intake is carefully regulated in patients with renal failure, and the use of salt substitutes will worsen hyperkalemia. 2 Increases in weight do need to be reported to the health-care provider as a possible indication of fluid volume excess, but this is not the reason why salt substitute is to be avoided. 3 An AV fistula does need to be protected from injury and infection could be caused by constricting clothing, venipunctures, and other items. 4 The control of hypertension is essential in the management of a client with kidney disease, but salt substitute is not known to interact with antihypertensive medications.

An older adult patient, who lives in a long-term care facility, presents in the emergency department (ED) due to fever, nausea, and vomiting over the past two days. The patient denies thirst. The urine dipstick indicates a decreased urine specific gravity. Which medical diagnosis should the nurse anticipate when planning care for this patient? 1) Dehydration 2) Hypertension 3) Fluid overload 4) Congestive heart failure

ans 1 Feedback 1 Older adult patients are less able to concentrate their urine, making them susceptible to dehydration. In addition, there is a deficit of the thirst response. However, fever, nausea, and vomiting resulting from these changes are not considered normal. The patient's symptoms of nausea and vomiting suggest decreased intake and increased output through vomiting, placing the client at risk for dehydration. 2 Hypertension does not manifest with the current clinical indicators. 3 Congestive heart failure and fluid overload would present with respiratory difficulty and peripheral edema. 4 Congestive heart failure and fluid overload would present with respiratory difficulty and peripheral edema

Which statement should the nurse make when communicating the "S" in the SBAR approach for effective communication? 1) "The patient presented to the emergency department at 0200 with lower left abdominal pain." 2) "The patient rated the pain upon admission as a 9 on a 10-point numeric scale." 3) "The patient has no significant issues in the medical history." 4) "The patient was given a prescribed opioid analgesic at 0300."

ans 1 Feedback 1 The "S" reflects the patient's current situation which is communicated by providing a brief statement of the issue. This statement by the nurse exemplifies the current situation. 2 The "A" reflects the patient's assessment data. This statement by the nurse exemplifies the patent's assessment data. 3 The "B" reflects the patient's medical history. This statement by the nurse exemplifies communicating the patient's history related to the current problem. 4 The "R" reflects specific actions needed to address the situation. This statement by the nurse exemplifies the actions implemented to address current level of pain.

The nurse is providing care for an adult patient who is admitted to the emergency department (ED) after passing out. The patient has been fasting and currently has ketones in the urine. Which acid-based imbalance should the nurse monitor the patient for based on the current data? 1) Metabolic acidosis 2) Metabolic alkalosis 3) Respiratory acidosis 4) Respiratory alkalosis

ans 1 Feedback 1 The patient who is fasting is at risk for metabolic acidosis. The body recognized fasting as starvation and begins to metabolize its own proteins into ketones, which are metabolic acid. 2 The nurse would not monitor this patient for metabolic alkalosis. 3 The nurse would not monitor this patient for respiratory acidosis. 4 The nurse would not monitor this patient for respiratory alkalosis.

Which is the priority nursing action for the ethical decision-making process? 1) Determine exactly what needs to be decided. 2) Formulate alternatives to solve the issue. 3) Implement an action to achieve the greatest benefit with the least amount of risk. 4) Ascertain if new information is available regarding the issue.

ans 1 Feedback 1 The priority action for the ethical decision-making process is assessment. During this step, the nurse determines exactly what needs to be decided. 2 During the planning stage of ethical decision-making, the nurse formulates alternatives to solve the issue. 3 During the implementation stage of ethical decision-making, the nurse implements an action to achieve the greatest benefit with the least amount of risk. 4 During the evaluation stage of ethical decision-making, the nurse ascertains if new information is available regarding the issue to determine if new actions should be implemented.

The nurse is planning care for an older adult patient with respiratory acidosis. Which intervention should the nurse include in this patient's plan of care? 1) Maintain adequate hydration. 2) Reduce environmental stimuli. 3) Administer intravenous sodium bicarbonate. 4. administer prescribed IV fluids carefully.

ans 1 In respiratory acidosis, there are a drop in the blood pH, reduced level of oxygen, and retaining of carbon dioxide. The body needs to be well-hydrated so that pulmonary secretions can be removed to improve oxygenation.

The nurse is instructing a patient with heart failure about a prescribed sodium-restricted diet. Which patient statement indicates that additional teaching is required? 1) "I can use as much salt substitute as I want." 2) "I have to read the labels on foods to find out the sodium content." 3) "I have to limit the intake of food with baking soda or baking powder." 4) "I can use spices and lemon juice to add flavor to food when cooking."

ans 1 Low-sodium salt substitutes are not really sodium-free. They may contain half as much sodium as regular salt. The patient should be instructed to use salt substitutes sparingly because larger amounts often taste bitter instead of salty

The nurse is reviewing the health-care provider orders for a patient who is diagnosed with respiratory alkalosis. Which prescription is appropriate for this patient's care needs? 1) Draw arterial blood gas analysis. 2) Administer oxygen via face mask. 3) Restrict fluids to two liters per day. 4) Infuse one ampule of sodium bicarbonate

ans 1 Management of respiratory alkalosis focuses on correcting the imbalance and treating the underlying cause. Arterial blood gases must be ordered prior to beginning medication or oxygen therapy.

The nurse is providing care to a patient who has been vomiting for several days. The nurse knows that the patient is at risk for metabolic alkalosis because gastric secretions have which characteristic? 1) Gastric secretions are acidic. 2) Gastric secretions are alkaline. 3) Gastric secretions have a foul smell. 4) Gastric secretions are green in color.

ans 1 Metabolic alkalosis due to loss of hydrogen ions usually occurs because of vomiting or gastric suction. Gastric secretions are highly acidic (pH 1-3). When these are lost through vomiting or gastric suction, the alkalinity of body fluids increases. This increased alkalinity results from the loss of acid and from selective retention of bicarbonate by the kidneys as chloride is depleted.

The nurse is providing care for a postpartum patient who states, "I know my rights and you have to do what I tell you!" Which response by the nurse is appropriate? 1) "I don't mind doing anything within reason, but you have a responsibility to be considerate to the staff as well." 2) "That statement is not included in your patient rights; don't yell at me." 3) "Why do you feel angry ... did I do something you did not like?" 4) "Do you want me to take the baby to the nursery so you can calm down?"

ans 1 Most hospitals now publish lists of patient responsibilities, emphasizing that health care is a partnership between the patient and caregivers, that other patients have a right to be comfortable too, and that there are consequences if patients don't comply with treatment plans, cooperate with the health-care team, or be considerate of the staff and other patients

The nurse managers in a community hospital have been charged with reviewing job descriptions of unlicensed assistive personnel (UAPs) and have questions about the delegation of certain patient care activities to UAPs by nurses. To which group, organization, or individual would committee members direct their questions to obtain definitive answers about the parameters of nurse delegation to UAPs? 1) The state board of nursing 2) The American Nurses Association 3) The hospital's Chief Nursing Officer 4) The hospital's Chief Executive Officer

ans 1 Parameters for the delegation of patient care tasks by nurses to UAPs are established by each state's board of nursing.

The medical-surgical nurse identifies a clinical practice issue and wants to determine if there is sufficient evidence to support a change in practice. Which type of study provides the strongest evidence to support a practice change? 1) Randomized control study 2) Quasi-experimental study 3) Case-control study 4) Cohort study

ans 1 Systematic reviews of randomized control studies (Level I) are the highest level of evidence because they include data from selected studies that randomly assigned participants to control and experimental groups. The lower the numerical rating of the level of evidence indicates the highest level of evidence; therefore, this type of study provides the strongest evidence to support a practice change

The nurse is analyzing the patient's arterial blood gas report, which reveals a pH of 7.15. The patient has just suffered a cardiac arrest. Which consequences of this pH value does the nurse consider for this patient? 1) Decreased cardiac output 2) Decreased potassium levels 3) Increased magnesium levels 4) Decreased free calcium in the ECF

ans 1 The nurse knows that severe acidosis depresses myocardial contractility, which leads to decreased cardiac output

A patient with metabolic acidosis has been admitted to the unit from the emergency department (ED). The patient is experiencing confusion and weakness. Which independent nursing intervention is the priority? 1) Protecting the patient from injury 2) Placing the patient in a high-Fowler's position 3) Administering sodium bicarbonate to the patient 4) Providing the patient with appropriate skin care

ans 1 The patient with metabolic acidosis may have symptoms of drowsiness, lethargy, confusion, and weakness. A priority of care would be preventing injury

3. The nurse is conducting a medication assessment for a preoperative patient. Which action by the nurse is appropriate for the patient who is prescribed amiodarone? 1) Obtaining a baseline ECG 2) Monitoring blood pressure 3) Assessing for hyperglycemia 4) Tapering the drug two days prior to surgery

ans 1 The prescribed drug is an antiarrhythmic; therefore, the most appropriate nursing action is to obtain a baseline ECG.

Which is the priority nursing action when providing care to a patient who is admitted with metabolic alkalosis? 1) Monitoring oxygen saturation 2) Setting goals for the plan of care 3) Administering prescribed medications 4) Teaching the family about risk factors

ans 1 The priority for this patient is monitoring oxygen saturation. The depressed respiratory drive that often accompanies metabolic alkalosis can lead to hypoxemia and impaired oxygenation of the tissues.

A local hospital formed a neurotrauma (NT) team with the following members: acute care nurses, physicians, other care partners (e.g., physical therapists, social workers, case managers, dieticians), and representatives from the NT outpatient clinic. This team is led by a physician who makes treatment decisions based on the treatment plans developed by individual team members who each communicate with the patients, asking the same or similar questions to obtain data needed for their treatment plan. Which type of communication and action is represented in the scenario described? 1) Parallel communication 2) Parallel functioning 3) Information exchange 4) Coordination and consultation

ans 1 The type of communication and action used by this health-care team is parallel communication. It is at the lowest level along the continuum of communication and collaboration among health team members and is characterized by each professional communicating with the patient independently, asking the same or similar questions needed to develop their plan of care.

The medical-surgical nurse is providing patient care. Which circumstance would necessitate the nurse verifying the patient's identification using at least two sources? 1) Prior to delivering a meal tray 2) Prior to passive range of motion 3) Prior to medication administration 4) Prior to documenting in the medical record

ans 1 Visitation rights should be evaluated prior to a TJC accreditation site visit as this aspect of patient-centered care is incorporated into the site evaluation.

The nurse is caring for a patient with congestive heart failure who is admitted to the medical-surgical unit with acute hypokalemia. Which prescribed medication may have contributed to the patient's current hypokalemic state? 1) Cortisol 2) Demerol 3) Skelaxin 4) Nonsteroidal anti-inflammatory drugs (NSAIDs

ans 1 excess potassium loss through the kidneys is often caused by such medications as corticosteroids, potassium-wasting (loop) diuretics, amphotericin B, and large doses of some antibiotics. Cortisol is a type of corticosteroid and can cause hypokalemia.

A patient tells the nurse, "I don't really like the nurse on the first shift; she treats me bad." Which action by the nurse is appropriate in order to advocate for this patient? 1) Call the agency patient advocacy department. 2) Confront the nurse when she comes to work. 3) Tell the patient he or she has the right to switch nurses. 4) Call the local authorities.

ans 1 individual patients who feel their rights have been violated or are endangered have a number of options. Many hospitals and large provider agencies have patient advocates who can help patients navigate the system and intervene to ensure that their rights are maintained.

22. The nurse is instructing a patient with heart failure about a prescribed sodium-restricted diet. Which patient statement indicates that additional teaching is required? 1) "I can use as much salt substitute as I want." 2) "I have to read the labels on foods to find out the sodium content." 3) "I have to limit the intake of food with baking soda or baking powder." 4) "I can use spices and lemon juice to add flavor to food when cooking."

ans 1 low-sodium salt substitutes are not really sodium-free. They may contain half as much sodium as regular salt. The patient should be instructed to use salt substitutes sparingly because larger amounts often taste bitter instead of salty.

The medical-surgical nurse wants to determine if a policy change is needed for an identified clinical problem. Which is the first action the nurse should implement? 1) Developing a question 2) Disseminating the findings 3) Conducting a review of the literature 4) Evaluating outcomes of practice change

ans 1 the first step of evidence-based practice is to develop a question based on the clinical issue.

Which statement regarding the use of the nursing process in clinical practice is accurate? 1) "The nursing process is closely related to clinical decision-making." 2) "The nursing process is used by all members of the interprofessional team to plan care." 3) "The nursing process has 4 basic steps: assessment, planning, implementation, evaluation." 4) "The nursing process is being replaced by the implementation of evidence-based practice."

ans 1 the nursing process is closely related to the nurse's decision-making in the clinical environment. This statement is accurate.

When the nurse receives a telephone order from the health-care provider's office, which guidelines are used to ensure the order is correct? Select all that apply. 1) Ask the prescriber to speak slowly. 2) Read the order back to the prescriber. 3) Know agency policy for telephone orders. 4) Sign the prescriber's name and credentials. 5) Ask the prescriber to repeat or spell out medication

ans 1,2,3,5 Feedback 1. This is correct. When receiving a telephone order from a provider, the nurse should ask the prescriber to repeat or spell out the medication, to speak slowly, and read the order back to the prescriber once the prescription is complete. 2. This is correct. When receiving a telephone order from a provider, the nurse should ask the prescriber to repeat or spell out the medication, to speak slowly, and read the order back to the prescriber once the prescription is complete. 3. This is correct. It is also important for the nurse to know the agency's policy regarding telephone orders. 4. This is incorrect. The nurse does not sign the prescriber's name and credentials; the nurse only transcribed the prescription and the prescriber countersigns it within a time period prescribed by the agency's policy. 5. This is correct. When receiving a telephone order from a provider, the nurse should ask the prescriber to repeat or spell out the medication, to speak slowly, and read the order back to the prescriber once the prescription is complete.

The nurse is providing care to a pregnant patient with a history of drug use. The patient refuses testing for human immunodeficiency virus (HIV) despite the recommendation of her nurse-midwife. Which actions by the nurse are appropriate in this situation? Select all that apply. 1) Refusing to treat the patient unless she is tested 2) Running the test without the patient's knowledge 3) Emphasizing the importance of the test to the patient 4) Offering counseling regarding the testing 5) Encouraging the patient to reconsider the decision to be tested throughout the pregnancy

ans 1,2,4 1. This is correct. Strikers may be concerned about patient care as it is related to adequate staffing. 2. This is corrans 1,2,4 1. This is correct. Strikers may be concerned about patient care as it is related to adequate staffing. 2. This is correct. Strikes may adversely affect patient care and outcomes. 3. This is incorrect. The desire to take time off and the need for higher pay are not ethical issues. 4. This is correct. Nurses may feel allegiance to a hospital where they have worked for years. 5. This is incorrect. The desire to take time off and the need for higher pay are not ethical issues.ect. Strikes may adversely affect patient care and outcomes. 3. This is incorrect. The desire to take time off and the need for higher pay are not ethical issues. 4. This is correct. Nurses may feel allegiance to a hospital where they have worked for years. 5. This is incorrect. The desire to take time off and the need for higher pay are not ethical issues.

The nurse manager wants to designate a member of the nursing team as the care coordinator for a patient who will require significant care during the hospitalization. Which skills should this nurse possess in order to assume this role? Select all that apply. 1) Effective clinical reasoning 2) Effective communication skills 3) Effective infection control procedures 4) Effective documentation 5) Effective intravenous skills

ans 1,2,4 Feedback 1. This is correct. The physician is a member of the interprofessional team and should be invited to participate in the care conference. 2. This is correct. The pharmacist is a member of the interprofessional team and should be invited to participate in the care conference. 3. This is incorrect. The unit secretary is not a member of the interprofessional team; therefore, would not require an invitation to attend the care conference. 4. This is correct. The social worker is a member of the interprofessional team; therefore, should be invited to participate in the care conference. 5. This is incorrect. The home care aide, while a member of the interprofessional team, would not benefit from attending a care conference while the patient is hospitalized.

Which are ethical issues for the nurse to consider prior to deciding whether or not to honor the picket line during a strike situation? Select all that apply. 1) The need to support coworkers in their efforts to improve working conditions 2) The need to ensure that clients receive care and are not abandoned 3) The desire to take some time off 4) Loyalty to the nurse's employer 5) The need for higher pay

ans 1,2,4 1. This is correct. Strikers may be concerned about patient care as it is related to adequate staffing. 2. This is correct. Strikes may adversely affect patient care and outcomes. 3. This is incorrect. The desire to take time off and the need for higher pay are not ethical issues. 4. This is correct. Nurses may feel allegiance to a hospital where they have worked for years. 5. This is incorrect. The desire to take time off and the need for higher pay are not ethical issues.

The hospital's nurse case manager has been extensively involved with a shooting victim and members of the patient's family in coordinating care of providers from many disciplines as the patient progressed from the emergency department (ED) to the intensive care unit (ICU), and then onto the medical-surgical unit. After three weeks of hospitalization, the case manager is helping to prepare the patient for discharge to a rehabilitation center where treatment will continue. Which outcomes have been documented in the literature as benefits of such collaboration? Select all that apply. 1) Improved patient outcomes 2) Decreased duplication of health-care services 3) Increased overall cost of health-care services 4) Decreased patient morbidity and mortality 5) Decreased level of job satisfaction

ans 1,2,4, Feedback 1. This is correct. Research findings suggest that collaboration in health care among patients, family members, caregivers, and communities leads to improved patient outcomes, a reduction in duplicated health-care services, and a decrease in patient morbidity and mortality. 2. This is correct. Research findings suggest that collaboration in health care among patients, family members, caregivers, and communities leads to improved patient outcomes, a reduction in duplicated health-care services, and a decrease in patient morbidity and mortality. 3. This is incorrect. Research findings suggest that collaboration in health care among patients, family members, caregivers, and communities leads to a decreased, not increased, cost of care. 4. This is in correct. Research findings suggest that collaboration in health care among patients, family members, caregivers, and communities leads to improved patient outcomes, a reduction in duplicated health-care services, and a decrease in patient morbidity and mortality. 5. This is incorrect. Collaborative efforts have also been found to contribute to an enhanced sense of autonomy. This increase in sense of autonomy has been linked to nurses' greater job satisfaction.

The nurse is concerned that an older adult patient is at risk for developing acute renal failure. Which information in the patient's history support the nurse's concern? Select all that apply. 1) Diagnosed with hypotension 2) Recent aortic valve replacement surgery 3) Total hip replacement surgery five years ago 4) Taking medication for type 2 diabetes mellitus 5) Prescribed high doses of intravenous antibiotic

ans 1,2,5 Feedback 1. This is correct. Older adults develop acute renal failure more frequently because of the higher incidence of serious illnesses, hypotension, major surgeries, diagnostic procedures, and treatment with nephrotoxic drugs. Decreased kidney function associated with aging also puts the older patient at risk for kidney failure. Hypotension, scheduled for aortic valve replacement surgery, and receiving high doses of intravenous antibiotics increase this patient's risk for developing acute renal failure. 2. This is correct. Older adults develop acute renal failure more frequently because of the higher incidence of serious illnesses, hypotension, major surgeries, diagnostic procedures, and treatment with nephrotoxic drugs. Decreased kidney function associated with aging also puts the older patient at risk for kidney failure. Hypotension, scheduled for aortic valve replacement surgery, and receiving high doses of intravenous antibiotics increase this patient's risk for developing acute renal failure. 3. This is incorrect. A previous history of major surgery and current treatment for type 2 diabetes mellitus are not identified risk factors for the development of acute renal failure. 4. This is incorrect. A previous history of major surgery and current treatment for type 2 diabetes mellitus are not identified risk factors for the development of acute renal failure. 5. This is correct. Older adults develop acute renal failure more frequently because of the higher incidence of serious illnesses, hypotension, major surgeries, diagnostic procedures, and treatment with nephrotoxic drugs. Decreased kidney function associated with aging also puts the older patient at risk for kidney failure. Hypotension, scheduled for aortic valve replacement surgery, and receiving high doses of intravenous antibiotics increase this patient's risk for developing acute renal failure

Which nursing actions are appropriate when conducting an Allen test? Select all that apply. 1) Rest the patient's arm on the mattress. 2) Support the patient's wrist with a rolled towel. 3) Tell the patient to relax the hand and then clench a fist. 4) Ensure that a second nurse is available to assist with the procedure. 5) Press the patient's radial and ulnar arteries using the index and middle fingers

ans 1,2,5 Feedback 1. This is correct. Rest the patient's arm on the mattress or bedside stand and support his wrist with a rolled towel. 2. This is correct. Rest the patient's arm on the mattress or bedside stand and support his wrist with a rolled towel. 3. This is incorrect. The nurse will tell the patient to first clench the fist, hold the position for a few seconds and then hold the hand in a relaxed position. 4. This is incorrect. A second nurse is not required to perform this test. 5. This is correct. The nurse uses the index and middle fingers to press on the patient's radial and ulnar arteries.

The community nurse visits the home of a young child who is home from school because of sudden onset of nausea, vomiting, and lethargy. The nurse suspects acute renal failure. Which clinical manifestations support the nurse's suspicions? Select all that apply. 1) Edema 2) Wheezing 3) Hematuria 4) Postural hypotension 5) Elevated blood pressure

ans 1,3,5 Feedback 1. This is correct. Pediatric manifestations of acute renal failure characteristically begin with a healthy child who suddenly becomes ill with nonspecific symptoms that indicate a significant illness or injury. These symptoms may include any combination of the following: nausea, vomiting, lethargy, edema, gross hematuria, oliguria, and hypertension. 2. This is incorrect. Wheezing is not a manifestation of acute renal failure. 3. This is correct. Pediatric manifestations of acute renal failure characteristically begin with a healthy child who suddenly becomes ill with nonspecific symptoms that indicate a significant illness or injury. These symptoms may include any combination of the following: nausea, vomiting, lethargy, edema, gross hematuria, oliguria, and hypertension. 4. This is incorrect. Postural hypotension is a manifestation of acute renal failure in an older person. 5. This is correct. Pediatric manifestations of acute renal failure characteristically begin with a healthy child who suddenly becomes ill with nonspecific symptoms that indicate a significant illness or injury. These symptoms may include any combination of the following: nausea, vomiting, lethargy, edema, gross hematuria, oliguria, and hypertension

A hospice nurse is providing care to a patient diagnosed with ovarian cancer. The patient is concerned that her two daughters are at an increased risk for cancer and asks the nurse for help. Which actions by the nurse are appropriate? Select all that apply. 1) Provide the family with information on hereditary cancer risks. 2) Assure the client that ovarian cancer is not hereditary. 3) Offer to refer the daughters to a genetic counselor. 4) Arrange for the client to have genetic testing. 5) Tell the client that her additional worrying is too stressful.

ans 1,3,5 Feedback 1. This is correct. A nurse's role as educator is crucial to ethical practice. 2. This is incorrect. Inaccurate reassurance or avoidance does not respect the patient's rights. 3. This is correct. Providing appropriate alternatives and options for the patient and the family are correct responses to the patient's concerns. 4. This is correct. Providing appropriate alternatives and options for the patient and the family are correct responses to the patient's concerns. 5. This is incorrect. Inaccurate reassurance or avoidance does not respect the patient's rights.

When discussing the importance of interprofessional collaboration, which advantages should the nurse include? Select all that apply. 1) Improved team member satisfaction 2) Increased division among team members 3) Increased safety with medication administration 4) Enhanced communication among team members 5) Increased patient satisfaction with discharge transition process

ans 1,4,5 Feedback 1. This is correct. Improved team member satisfaction is an advantage of interprofessional collaboration. 2. This is incorrect. There is a decreased, not increased, division among team members with interprofessional collaboration. 3. This is incorrect. There is increased safety with the discharge transition process, not medication administration, with interprofessional collaboration. 4. This is correct. Enhanced communication among team members is an advantage of interprofessional collaboration. 5. This is correct. Increased patient satisfaction with the discharge transition process is an advantage of interprofessional collaboration.

The nurse is caring for a patient who is receiving intravenous fluids postoperatively following cardiac surgery. The nurse is aware that this patient is at risk for fluid volume excess. The family asks why the patient is at risk for this condition. Which response by the nurse is the most appropriate? 1) "Fluid volume excess is caused by inactivity." 2) "Fluid volume excess is caused by the intravenous fluids." 3) "Fluid volume excess is caused by new onset liver failure caused by the surgery." 4) "Fluid volume excess is common due to increased levels of antidiuretic hormone in response to the stress of surgery."

ans 17 antidiuretic hormone (ADH) and aldosterone levels are commonly increased following the stress response before, during, and immediately after surgery. This increase leads to sodium and water retention. Adding more fluids intravenously can cause a fluid volume excess and stress upon the heart and circulatory system.

Which identifier should the nurse use during the initial time-out to determine the right patient? 1) Date of birth 2) Maiden name 3) Medical record number 4) Photo placed in the medical record

ans 1Date of birth is an identifier the nurse should use to determine the right patient during the initial time-out conducted during the preoperative period

Which statement regarding the American Nurses Association (ANA) Code of Ethics for professional nurses is accurate? 1) "It is used by all health-care professionals." 2) "It guides nurses in their professional behavior and relationships." 3) "It forms the basis for possible lawsuits." 4) "It is the only code of ethics available for nurses."

ans 2 Feedback 1 Each profession has its own code of ethics. 2 The ANA Code of Ethics is a guide for nurses in their work with clients and other professionals. 3 State laws regarding nursing are the basis of lawsuits, not the Code of Ethics. 4 There is also an International Code of Ethics promulgated by the International Council of Nurses

Which is a basic principle of the Patient Protection and Affordable Care Act of 2010 that the nurse should include in a teaching session for members of the health-care team? 1) Decreased access 2) Decreased cost of care 3) Decreased quality of care 4) Decreased safety

ans 2 Feedback 1 Increased, not decreased, access is a basic principle of the Patient Protection and Affordable Care Act of 2010. 2 Decreased cost of care is a basic principle of the Patient Protection and Affordable Care Act of 2010. 3 Increased, not decreased, quality of care is a basic principle of the Patient Protection and Affordable Care Act of 2010. 4 Increased, not decreased, safety is a basic principle of the Patient Protection and Affordable Care Act of 2010.

The nurse is providing care to an older adult patient with terminal cancer who has opted to discontinue treatment and go home. The patient's family, however, wants to continue treatment. The nurse agrees to be present while the patient tells the family. Which ethical patient principle is the nurse supporting? 1) Beneficence 2) Autonomy 3) Nonmaleficence 4) Justice

ans 2 Feedback 1 Beneficence means "doing good." 2 Autonomy refers to the right to make one's own decisions. The nurse is supporting this principle by supporting the client in his decision. 3 Nonmaleficence is the duty to "do no harm." 4 Justice is often referred to as fairness.

Which statement best describes the American Nurses Association (ANA) Code of Ethics for professional nurses? 1) "It alleviates suffering for those cared for by professional nurses." 2) "It provides standards for professional nursing practice." 3) "It reflects legal judgments in professional nursing practice." 4) "It serves as legal standards for professional nursing practice."

ans 2 Feedback 1 Codes of ethics provide the atmosphere in which the nurse is able to alleviate suffering. 2 The ANA Code of Ethics is a formal statement of the group's ideals and values. It is a set of ethical principles that serves as a standard for professional actions. 3 Codes of ethics do not necessarily reflect legal judgments. 4 Codes of ethics usually have higher requirements than legal standards, and they are never lower than the legal standards of the profession.

The results of a patient's arterial blood gas sample indicate an oxygen level of 72 mmHg. Which should the nurse closely assess when providing care to this patient? 1) Perfusion 2) Cognition 3) Communication 4) Fluid and electrolytes

ans 2 An oxygen level of less than 75 mmHg can be due to hypoventilation. This drop in oxygen will change the patient's level of responsiveness.

The nurse is preparing to administer diphenhydramine to a patient who is experiencing a severe allergic reaction to peanuts. Which information about the drug should the nurse provide to the patient? 1) "This is the medication of choice to treat airway obstruction." 2) "This medication will help relieve your itching and runny nose." 3) "This medication will prevent you from going into anaphylactic shock." 4) "This medication will take a while to be effective but will control your symptoms for several hours."

ans 2 Antihistamines help to relieve histamine-related symptoms such as itching, flushing, hives, and rhinorrhea.

Which patient statement indicates the need for additional education regarding the use of sodium bicarbonate to treat acidosis? 1) "I need to purchase antacids without salt." 2) "I should use the antacid for at least 2 months." 3) "I should contact the doctor if I have any gastric discomfort with chest pain." 4) "I should call the doctor if I get short of breath or start to sweat with this medication."

ans 2 Bicarbonate antacid should not be used for longer than two weeks. This statement indicates the need for additional teaching.

The nurse is providing care to a patient who seeks emergency treatment for headache and nausea. The patient works in a mill without air conditioning. The patient states, "I drink water several times each day but I seem to sweat more than I am able to replace." Which suggestions should the nurse provide to this patient? 1) Drink juices and carbonated sodas. 2) Eat something salty when drinking water. 3) Eat something sweet when drinking water. 4) Double the amount of water being ingested

ans 2 Both salt and water are lost through sweating. When only water is replaced, the individual is at risk for salt depletion. Symptoms include fatigue, weakness, headache, and gastrointestinal symptoms such as loss of appetite and nausea. The client should be instructed to eat something salty when drinking water to help replace the loss of sodium.

An older adult patient, who appears intermittently confused, is admitted to the hospital after a fall. Based on the current data, which is the patient at an increased risk for developing? 1) Brain attack 2) Dehydration 3) Hemorrhage 4) Kidney damage

ans 2 During the aging process, the thirst mechanism declines. In a patient with an altered level of consciousness, this can increase the risk of dehydration and high serum osmolality.

The nurse is providing care to a client who is considered brain dead. The family has opted to end care and the health-care provider asks the nurse to pull the endotracheal (ET) tube. The nurse is uncomfortable with this request. Which is the reason the nurse is experiencing difficulty with this task? 1) An ethical conflict 2) Personal values 3) Legal issues 4) A cultural conflict

ans 2 Feedback 1 The decision is within ethical principles. 2 The nurse is distressed because of personal values, which are in conflict with causing the client's death. 3 Extubating this patient would not be a legal decision. 4 Cultural values are not evidenced in this instance.

Which statement is a primary and historical barrier to effective nurse-physician collaboration that has persisted over time? 1) The view among the general population that nurses' contributions to patients' care is less important to their health and well-being compared to the contribution of physicians 2) The nurses' and physicians' perceptions of inequity in their roles, with nurses assuming a subservient role and physicians assuming leadership and superior role in health-care settings 3) A general lack of education provided in schools for health professionals about the benefits on health-care quality linked 4) A lack of published evidence about the effectiveness of collaborative efforts among and between nurses and physicians to nurse-physician collaboration

ans 2 Feedback 1 The decision is within ethical principles. 2 The nurse is distressed because of personal values, which are in conflict with causing the client's death. 3 Extubating this patient would not be a legal decision. 4 Cultural values are not evidenced in this instance.

Which patient population should the nurse focus on to increase access to care that is coordinated, safe, and focused on the patient's unique needs across all care settings? 1) Pediatric patients 2) Older adult patients 3) Young adult patients 4) Acute needs patients

ans 2 Feedback 1 The pediatric patient population is not identified as a group where access to coordinated, safe, and focused care is lacking across care settings. 2 Access to care that is coordinated, safe, and focused on the patient's unique needs across all care settings has eluded many patients, particularly the elderly and chronically ill. 3 The young adult patient population is not identified as a group where access to coordinated, safe, and focused care is lacking across care settings. 4 Patients requiring acute care is not identified as a group where access to coordinated, safe, and focused care is lacking across care settings

The nurse is providing care to a patient whose serum calcium levels have increased since a surgical procedure performed three days prior. Which intervention should the nurse implement to decrease the risk for the development of hypercalcemia? 1) Monitor vital signs every eight hours 2) Encourage ambulation three times a day 3) Irrigate the Foley catheter one time a day 4) Recommend turning, coughing, and deep breathing every two hours

ans 2 Feedback 1 This intervention is not appropriate to decrease the risk for the development of hypercalcemia. 2 Hypercalcemia can occur from immobility. Ambulation of the client helps to prevent leaching of calcium from the bones into the serum. 3 This intervention is not appropriate to decrease the risk for the development of hypercalcemia. 4 This intervention is not appropriate to decrease the risk for the development of hypercalcemia.

The nurse is providing care to a patient who is intubated and receiving mechanical ventilation after a motor vehicle crash. The patient is fighting the ventilator and attempting to remove the endotracheal tube. Which nursing action decreases the patient's risk for developing respiratory alkalosis? 1) Apply wrist restraints. 2) Administer a prescribed sedative. 3) Teach the patient to take slow, deep breaths. 4) Discuss removing the endotracheal tube with the health-care provider.

ans 2 For a patient being mechanically ventilated, the only way to reduce rapid respirations might be to provide a sedative.

Which should be the focus of an educational session for nurses and other members of the interdisciplinary team when addressing high rates of patient readmission to the health system? 1) Medication errors 2) Coordination of care 3) Adverse clinical events 4) Roles of each member providing care

ans 2 Hospital readmission rates are often attributed to a lack of coordination of care as patients are discharged to rehabilitation facilities, long-term care agencies, or back to their homes; therefore, this should be the focus of the educational session.

Which aspect of patient-centered care should the nurse manager evaluate prior to The Joint Commission site visit for accreditation? 1) Visitation rights 2) Education level of staff 3) Fall prevention protocol 4) Infection control practices

ans 2 Implementation of patient-centered care is the benchmark in which acute care facilities are evaluated against.

The nurse is caring for a patient who is reporting pain of 8/10 on a 1 to 10 numeric pain scale. The nurse administers the prescribed pain medication. When the nurse re-evaluates the patient one hour later, the patient is still reporting pain of 8/10. Which action by the nurse is appropriate at this time? 1) Wait for the health-care provider to make rounds to report the problem.2) Report to the health-care provider by telephone. 3) Increase the dosage of the medication. 4) Include in the nursing report that the medication is ineffective.

ans 2 In this case reporting to the physician by telephone is appropriate

Handoff communication, the transfer of information during transitions in care such as during change-of-shift report, includes an opportunity to ask questions, clarify, and confirm the information between sender and receiver. Which is the main objective for ensuring effective communication during a patient handoff? 1) To avoid lawsuits 2) To ensure patient safety 3) To facilitate quality improvement 4) To make sure all documentation is done

ans 2 Ineffective communication is the primary cause of sentinel events, making patient safety the primary objective of the handoff communication process

A school-age patient is admitted to the pediatric intensive care unit (PICU), unconscious and with multiple traumatic injuries, after a skateboard accident that included a closed head injury. Many health professionals are involved in the patient's care and the scene is chaotic. The parents are extremely anxious and want to know what is happening. The case manager asks for an interdisciplinary team meeting to speak with the patient's parents. Which is the rationale for this meeting? 1) To allow for each specialty to practice independently 2) To share and evaluate information for care planning and implementation, and prevent priority conflicts, redundancy, and omissions in care 3) To all the primary health-care provider to make all the decision regarding the patient's care 4) To prevent the parents from trying to change the plan of care

ans 2 Interdisciplinary collaboration engages each professional's contribution to joint care planning, implementation, and accomplishment of patient goals, with possibly less redundancy, more efficiency, and fewer care omissions. The parents of a minor child should be involved in all aspects of care and decision making

A patient with acute renal failure has jugular vein distention, lower extremity edema, and elevated blood pressure. Based on this data, which nursing diagnosis is the most appropriate? 1) Risk for Infection 2) Excess Fluid Volume 3) Ineffective Renal Tissue Perfusion 4) Risk for Altered Cardiac Perfusion

ans 2 Jugular vein distention, edema, and elevated blood pressure are indications of excessive fluid. The diagnosis Excess Fluid Volume should be selected to guide this patient's care

The nurse is reviewing laboratory values for a female patient suspected of having a fluid imbalance. Which laboratory value evaluated by the nurse supports the diagnosis of dehydration? 1) Hematocrit 30% 2) Hematocrit 53% 3) Serum potassium 3.8 mEq/L 4) Serum osmolality 230 mOsm/kg

ans 2 The hematocrit measures the volume of whole blood that is composed of RBCs. Because the hematocrit is a measure of the volume of cells in relation to plasma, it is affected by changes in plasma volume. The hematocrit increases with severe dehydration.

The nurse is completing the preoperative checklist on the night shift in preparation for the patient's surgery, scheduled for 0800. Which tasks could the nurse complete at this time? 1) Documenting the time of last voiding 2) Checking the medical record for the history, physical, and signed informed consent 3) Administering preoperative medication 4) Removing the prosthesis

ans 2 The nurse on night shift could check the medical record to ensure that a history and physical have been completed, and that the consent for surgery is signed.

The nurse completes discharge teaching for a patient with an anxiety disorder. Which patient statement indicates correct understanding of information related to respiratory alkalosis? 1) "I will eat more bananas at breakfast." 2) "I will see my counselor on a regular basis." 3) "I will not take antacids when I have heartburn." 4) "I will breathe faster when I am feeling anxious."

ans 2 The patient understands that reducing anxiety can reduce hyperventilation and respiratory alkalosis. Seeing a counselor can help the patient develop alternative strategies for dealing with anxiety.

A patient with Type 1 diabetes mellitus has developed an open sore on the shin and is having trouble meeting daily goals for exercising. The patient is scheduled for discharge in a couple of days. When planning for this patient's continued care, who will the nurse notify regarding the patient's needs after discharge? 1) The pharmacy 2) The case manager 3) The physical therapist 4) The occupational therapist

ans 2 The patient's needs and progress have changed. The nurse notifies the case manager to coordinate changes in care needed after discharge. This patient's exercise program needs to be revamped, and the case manager is the individual to coordinate this change.

. The nurse receives shift report on a pediatric medical-surgical unit. The nurse has been assigned four patients for the shift. Which child does the nurse plan to assess first based on the increased risk for dehydration? 1) A 4-year-old child with a broken leg 2) A 15-month-old child with tachypnea 3) A 16-year-old child with migraine headaches 4) A 10-year-old child with cellulitis of the left leg

ans 2 The pediatric patient with the greatest risk for dehydration is the child who is under 2 years of age experiencing tachypnea which increases insensible fluid loss.

The nurse receives shift report on a pediatric medical-surgical unit. The nurse has been assigned four patients for the shift. Which child does the nurse plan to assess first based on the increased risk for dehydration? 1) A 4-year-old child with a broken leg 2) A 15-month-old child with tachypnea 3) A 16-year-old child with migraine headaches 4) A 10-year-old child with cellulitis of the left leg

ans 2 The pediatric patient with the greatest risk for dehydration is the child who is under 2 years of age experiencing tachypnea which increases insensible fluid loss.

The nurse is providing care to a patient who has an increased number of lymphocytes. Which explanation should the nurse provide to the patient regarding this abnormality? 1) "An elevated neutrophil count indicates your body is battling a parasitic infection." 2) "An elevated neutrophil count indicates your body is battling a bacterial infection." 3) "An elevated neutrophil count indicates your body is experiencing an allergic reaction." 4) "An elevated neutrophil count indicates your body is experiencing an adaptive immune response."

ans 2 a bacterial infection is often indicated by an elevated neutrophil count.

The nurse is providing care to a patient who seeks emergency treatment for headache and nausea. The patient works in a mill without air conditioning. The patient states, "I drink water several times each day but I seem to sweat more than I am able to replace." Which suggestions should the nurse provide to this patient? 1) Drink juices and carbonated sodas. 2) Eat something salty when drinking water. 3) Eat something sweet when drinking water. 4) Double the amount of water being ingested.

ans 2 both salt and water are lost through sweating. When only water is replaced, the individual is at risk for salt depletion. Symptoms include fatigue, weakness, headache, and gastrointestinal symptoms such as loss of appetite and nausea. The client should be instructed to eat something salty when drinking water to help replace the loss of sodium

The home care nurse is planning care for a diabetic patient requiring an extensive dressing change twice a day, assistance with activities of daily living (ADLs), and comprehensive education. Which role is the nurse assuming by coordinating the care this patient requires? 1) Collaborator 2) Case manager 3) Health educator 4) Health promoter

ans 2 case management involves one or more individuals overseeing the needs and requirements of a particular individual's health

The nurse is providing care to an older adult patient who is receiving intravenous (IV) fluids at 150 mL/hr. The patient is currently exhibiting crackles in the lungs, shortness of breath, and jugular vein distention. Which complication of IV fluid therapy does the nurse suspect the patient is experiencing? 1) Speed shock 2) Fluid volume excess 3) Anaphylactic reaction 4) Pulmonary embolism

ans 2 fluid volume excess may occur when older adult patients receive intravenous fluids rapidly.

The nurse is providing care to a patient whose serum calcium levels have increased since a surgical procedure performed three days prior. Which intervention should the nurse implement to decrease the risk for the development of hypercalcemia? 1) Monitor vital signs every eight hours 2) Encourage ambulation three times a day 3) Irrigate the Foley catheter one time a day 4) Recommend turning, coughing, and deep breathing every two hour

ans 2 hypercalcemia can occur from immobility. Ambulation of the client helps to prevent leaching of calcium from the bones into the serum

A patient with acute renal failure has jugular vein distention, lower extremity edema, and elevated blood pressure. Based on this data, which nursing diagnosis is the most appropriate? 1) Risk for Infection 2) Excess Fluid Volume 3) Ineffective Renal Tissue Perfusion 4) Risk for Altered Cardiac Perfusion

ans 2 jugular vein distention, edema, and elevated blood pressure are indications of excessive fluid. The diagnosis Excess Fluid Volume should be selected to guide this patient's care.

The school nurse is preparing a class session for high school students on ways to maintain fluid balance during the summer months. Which interventions should the nurse recommend Select all that apply. 1) Drink diet soda. 2) Reduce the intake of coffee and tea. 3) Drink more fluids during hot weather. 4) Drink flat cola or ginger ale if vomiting. 5) Exercise during the hours of 10 am and 2 pm.

ans 2,3,4 Feedback 1. This is incorrect. Diet soda often contains caffeine. 2. This is correct. Actions to prevent fluid volume deficit during the summer months include increasing fluid intake, drinking flat cola or ginger ale if vomiting, and reducing the intake of coffee and tea. 3. This is correct. Actions to prevent fluid volume deficit during the summer months include increasing fluid intake, drinking flat cola or ginger ale if vomiting, and reducing the intake of coffee and tea. 4. This is correct. Actions to prevent fluid volume deficit during the summer months include increasing fluid intake, drinking flat cola or ginger ale if vomiting, and reducing the intake of coffee and tea. 5. This is incorrect. Exercising between the hours of 10 am and 2 pm, considered the hottest time of the day, should be avoided

The nurse is caring for the patient experiencing hypovolemic shock and metabolic acidosis. Which nursing actions are appropriate for this patient? Select all that apply. 1) Limit the intake of fluids. 2) Administer sodium bicarbonate. 3) Monitor ECG for conduction problems. 4) Keep the bed in the locked and low position. 5) Monitor weight on admission and discharge

ans 2,3,4 Feedback 1. This is incorrect. The treatment for hypovolemic shock would include the administration of fluids, not limiting fluids. 2. This is correct. Administering sodium bicarbonate and monitoring ECGs are appropriate for the patient with shock. 3. This is correct. Administering sodium bicarbonate and monitoring ECGs are appropriate for the patient with shock. 4. This is correct. The patient recovering from hypovolemic shock is at risk for injury, so the bed should be kept in the locked and low position. 5. This is incorrect. Patients being treated for hypovolemia will require daily weights, not a weight on admission and then discharge.

Which actions by the nurse enhance patient safety during medication administration? Select all that apply. 1) Answering the call bell while transporting medications for a different patient 2) Identifying the patient using two sources prior to administering the medication 3) Holding a medication if the patient's diagnosis does not support its use 4) Administering the medication two hours after the scheduled time 5) Having another nurse verify the prescribed dose of insulin the patient is to receive

ans 2,3,4 Feedback 1. This is incorrect. Hand-off communication is not required prior to the administration of medication. The nurse would, however, verify the patient's identity using two sources. 2. This is correct. Hand-off communication is required when patient care is transferred from one provider to another, such as during the change of shift. 3. This is correct. Hand-off communication is required when patient care is transferred from one provider to another, such as when a patient is transferred to the surgical suite. 4. This is correct. Hand-off communication is required when patient care is transferred from one provider to another, such as anytime the nurse receives a new patient assignment. 5. This is incorrect. Hand-off communication is not required prior to family visitation.

The nurse is planning an interprofessional care conference for a patient who is approaching discharge from the hospital. Which members of the interprofessional team should the nurse invite to attend? Select all that apply. 1) Physician 2) Pharmacist 3) Unit secretary 4) Social worker 5) Home care aide

ans 2,3,5 Feedback 1. This is incorrect. Monitor alarms should be audible even during family visitation. Inaudible alarms may impede patient safety. 2. This is correct. The nurse should assess the alarm parameters, comparing to the prescribed settings, at the start of each shift. This action enhanced patient safety. 3. This is correct. The nurse should respond to all alarms in a timely fashion, which enhances patient safety. 4. This is incorrect. Monitor alarms should be audible at all times, even when the patient is asleep to enhance patient safety. 5. This is correct. The nurse should adjust alarm parameters based on specific practitioner prescriptions. This action enhances safety.

The medical-surgical nurse assumes care for a patient who is receiving continuous cardiopulmonary monitoring. Which actions by the nurse enhance safety for this patient? Select all that apply. 1) Silencing the alarm during family visitation 2) Assessing the alarm parameters at the start of the shift 3) Responding to the alarm in a timely fashion 4) Decreasing the alarm volume to enhance restful sleep 5) Adjusting alarm parameters based on specified practitioner prescription

ans 2,3,5 Feedback 1. This is incorrect. Interruptions should be minimized during the medication administration process; therefore, the nurse should not answer the call bell for another patient while transporting medications for administration. 2. This is correct. Verification of the right patient is one of the rights of medication administration; therefore, the nurse would identify the patient using two sources prior to the administration of medication. 3. This is correct. The nurse should ensure that the rationale for all medications are associated with the patient condition; therefore, this action enhances patient safety during medication administration. 4. This is incorrect. One of the rights of medication administration is the right time, which correlates to 30 minutes before or 30 minutes after the scheduled time. This nursing action would not enhance patient safety during medication administration. 5. This is correct. Verifying the dose of a high-risk medication, such as insulin, enhances patient safety during medication administration.

A patient's serum sodium level is 150 mg/dL. Based on this data, which interventions should the nurse plan for this patient? Select all that apply. 1) Elevate the head of the bed. 2) Instruct on a low-sodium diet. 3) Monitor heart rate and rhythm. 4) Administer diuretics as prescribed. 5) Administer potassium supplement as prescribed.

ans 2,4, Feedback 1. This is incorrect. Elevating the head of the bed would be appropriate if the patient were demonstrating signs of fluid volume overload. This is not known at this time and would not be a routine intervention with an elevated sodium level. 2. This is correct. For an elevated sodium level, the electrolyte will need to be restricted, in the form of a low-sodium diet. 3. This is incorrect. Monitoring of heart rate and rhythm would be more appropriate with a potassium imbalance. 4. This is correct. Diuretics will remove excess fluid being held in the body because of the extra sodium. 5. This is incorrect. A potassium imbalance is not associated with a sodium imbalance. More information is needed before this intervention would be planned or implemented

The nurse is providing care to a patient who is admitted with manifestations of metabolic alkalosis. Which diagnostic test findings support the admitting diagnosis? Select all that apply. 1) Serum glucose level 142 mg/dL 2) Blood pH 7.47 and bicarbonate 34 mEq/L 3) Intravenous pyelogram shows kidney stones 4) Bilateral lower lobe infiltrates noted on chest x-ray 5) Electrocardiogram changes consistent with hypokalemia

ans 2,5 Feedback 1. This is incorrect. Serum glucose level is not used to confirm the diagnosis of metabolic alkalosis. 2. This is correct. In metabolic alkalosis, the blood pH will be greater than 7.45 and the bicarbonate level greater than 28 mEq/L. 3. This is incorrect. The presence of kidney stones is not associated with the development of metabolic alkalosis. 4. This is incorrect. The presence of bilateral lower lobe infiltrates on chest x-ray would not contribute to the development of metabolic alkalosis. This finding might be the result of metabolic alkalosis if the client's respiratory status is compromised. 5. This is correct. The ECG pattern shows changes similar to those seen with hypokalemia

The Chief Nursing Officer and Chief Medical Officer in an urban teaching hospital are leading a series of meetings with nurses, physicians, hospital lawyers, and risk managers to review and update hospital privileging procedures and requirements for advanced practice RNs and physicians new to the hospital. This is an example of which type of collaborative team? 1) Intradisciplinary 2) Interdisciplinary 3) Multidisciplinary 4) Complementary

ans 2A team comprising members from different disciplines that is focused on achieving a common goal is an interdisciplinary team. Their varying professional backgrounds helps to ensure that other perspectives are represented as the issue is considered.

The nurse is providing care to a 3-year-old child whose parents decide to decline further treatment for cancer, which has metastasized. There is a conflict between the child's parents and the rest of the family. Which should the nurse consider when determining the appropriate action for this patient? 1) The age of the child 2) The beliefs of the child 3) The values of the parents 4) The values of the rest of the family

ans 3 Feedback 1 The age of the child is not a relevant factor in the decision making if the child is under 18 years. 2 The child is too young to have values and beliefs. 3 When confronted with a conflict regarding care, one of the first actions by the nurse is to consider the values and beliefs of the parents who are making the decision. 4 The nurse is respectful with the rest of the family but should consider the parents' decision only

Which intervention should the nurse implement for a patient whose serum phosphorus level is 2.0 mg/dL? 1) Enforce contact precautions 2) Strain all urine for kidney stones 3) Encourage consumption of milk and yogurt 4) Discourage the consumption of a high-calorie diet

ans 3 A phosphorus level of 2.0 is low, and the client will need additional dietary phosphorus. Providing phosphorus-rich foods such as milk and yogurt is a good way to provide that additional phosphorus

The nurse is providing care to a patient who states, "My doctor is refusing to treat me because I am noncompliant with his recommendations." Which is the priority nursing action in this situation? 1) Have the patient contact a consumer agency. 2) Advise the patient to sue the health-care provider. 3) Take the patient's issue to the hospital ethics committee. 4) Notify the health-care provider of the patient's complaints

ans 3 Feedback 1 A consumer agency is not appropriate because this is an ethical matter. 2 The nurse never advises a patient to sue but assists the patient to find help resolving the issue. 3 Acting as a patient advocate and protecting the patient's rights, the nurse should enlist the help of the hospital ethics committee. 4 The nurse should act on behalf of the patient, and the best way to do that is by taking the issue to the hospital ethics committee, not to the health-care provider

Which action is appropriate when dealing with an ethical dilemma in practice? 1) Relying on nursing judgment 2) Examining all conflicts in the situation 3) Investigating all aspects of the situation 4) Making a decision based on the policy of the agency

ans 3 Feedback 1 Overconfidence can lead to poor decision making. 2 Examining the conflicts surrounding the issue is only one aspect of the situation to consider. 3 To avoid making a premature decision, the nurse plans to investigate all aspects of the dilemma before deciding. 4 Reading the agency policy regarding the matter addresses only one aspect of the situation

Which laboratory test should the postanesthesia care nurse monitor closely for a patient who is prescribed warfarin in the treatment of atrial fibrillation? 1) Serum glucose 2) Serum potassium 3) Prothrombin (PT) time 4) Blood urea nitrogen (BUN)

ans 3 A PT time is monitored closely for any patient who is prescribed warfarin. Warfarin is often stopped for several days prior to a surgical procedure. However, this patient will continue to be at an increased risk for bleeding.

The nurse manager is preparing a medical-surgical unit for The Joint Commission (TJC) visit With the nurse manager presenting staff education focusing on TJC benchmarks, which of the following topics would be most appropriate? 1) Implementation of evidence-based practice 2) Implementation of patient-centered care 3) Implementation of medical asepsis practices 4) Implementation of interprofessional care

ans 3 A lack of time with members of the health care team is a common theme of patient dissatisfaction.

Which is a common theme regarding patient dissatisfaction related to care provided in the hospital setting? 1) Space in hospital rooms 2) Medications received to treat pain 3) Time spent with the health-care team 4) Poor quality food received from dietary

ans 3 A lack of time with members of the health care team is a common theme of patient dissatisfaction.

Which intervention should the nurse implement for a patient whose serum phosphorus level is 2.0 mg/dL? 1) Enforce contact precautions 2) Strain all urine for kidney stones 3) Encourage consumption of milk and yogurt 4) Discourage the consumption of a high-calorie diet

ans 3 A phosphorus level of 2.0 is low, and the client will need additional dietary phosphorus. Providing phosphorus-rich foods such as milk and yogurt is a good way to provide that additional phosphorus.

The nurse is providing care to a patient who is prescribed furosemide as part of the treatment for congestive heart failure (CHF). The patient's serum potassium level is 3.4 mEq/L. Which food should the nurse encourage the patient to eat based on this data? 1) Peas 2) Iced tea 3) Bananas 4) Baked fish

ans 3 A potassium level of 3.4 is low, so the client should be encouraged to consume potassium-rich foods. Of the foods listed, the highest in potassium is banana.

the hospital with sudden, severe abdominal pain. Which arterial blood gas supports the patient's current diagnosis of respiratory alkalosis? 1) pH is 7.35 and PaO2 is 88. 2) pH is 7.30 and HCO3 is 30. 3) pH is 7.47 and PaCO2 is 25. 4) pH is 7.33 and PaCO2 is 36.

ans 3 Acute pain usually causes hyperventilation, which causes the CO2 to drop and the client to experience respiratory alkalosis. The pH would denote alkalosis and would be higher than 7.45. HCO3 would trend downward as the kidneys begin to compensate for the alkalosis by excreting HCO3. The PaO2 is likely to be normal unless the client has been hyperventilating for a long time and is beginning to tire

The nurse is providing care to a patient who is admitted to the emergency department with symptoms of a myocardial infarction (MI). Which is the primary purpose of the interventions administered to this patient? 1) Providing pain relief 2) Preventing extension of damage 3) Preventing cardiogenic shock 4) Reducing blood pressure

ans 3 Cardiogenic shock is the cause of death for many persons who have a myocardial infarction. Interventions are designed to reduce the risk of cardiogenic shock when treating a patient experiencing an MI.

Which chronic lung condition noted in the patient's health history supports the current diagnosis of respiratory acidosis? 1) Aspiration 2) Pneumonia 3) Cystic fibrosis 4) Hyperthyroidism

ans 3 Chronic lung disease such as asthma and cystic fibrosis puts the patient at risk for respiratory acidosis.

The nurse is teaching a group of children and their parents about the prevention of heat-related illness during exercise. Which statement by a parent indicates an appropriate understanding of the preventive techniques taught during the teaching session? 1) "My child only needs to hydrate at the end of an exercise session." 2) "Water is the drink of choice to replenish fluids that are lost during exercise." 3) "I will have my child stop every 15-20 minutes during the activity for fluids." 4) "It is important for my child to wear dark clothing while exercising in the heat.

ans 3 During activity, stopping for fluids every 15-20 minutes is recommended.

The nurse is teaching a group of children and their parents about the prevention of heat-related illness during exercise. Which statement by a parent indicates an appropriate understanding of the preventive techniques taught during the teaching session? 1) "My child only needs to hydrate at the end of an exercise session." 2) "Water is the drink of choice to replenish fluids that are lost during exercise." 3) "I will have my child stop every 15-20 minutes during the activity for fluids." 4) "It is important for my child to wear dark clothing while exercising in the heat."

ans 3 During activity, stopping for fluids every 15-20 minutes is recommended.

The staff nurse is teaching a group of student nurses the situations that necessitate hand-off communication. Which student responses indicate the need for further education related to this procedure? Select all that apply. 1) "A hand-off is required prior to administering a medication." 2) "A hand-off is required during change of shift." 3) "A hand-off is required for a patient is transferred to the surgical suite." 4) "A hand-off is required whenever the nurse receives a new patient assignment." 5) "A hand-off is required prior to family visitation."

ans 3 Effective staff communication is essential to safe patient care, especially during hand offs. Patient transfer to another unit of the hospital necessitate a change in who is responsible for direct patient care; therefore, this situation would necessitate the need for SBAR during the hand-off process.

A patient is diagnosed with a sexually transmitted infections (STI) and states to the nurse, "Promise you will not tell anyone about my condition." Which action should the nurse take, when considering the Health Insurance Portability and Accountability Act (HIPAA) of 1996? 1) Honor the patient's wishes 2) Respect the patient's privacy and confidentiality. 3) Communicate only necessary information. 4) Not disclosing any information to anyone.

ans 3 Feedback 1 Clients must be able to trust that their information is secure and will only be shared with appropriate entities. In this case, the nurse may be required to report information to the state health department. 2 Clients must be able to trust that their information is secure and will only be shared with appropriate entities. In this case, the nurse may be required to report information to the state health department. 3 HIPAA includes standards that protect the confidentiality, integrity, and availability of data as well as standards that define appropriate disclosures of identifiable health information and client rights protection. Nurses are entrusted with sensitive information, which at times must be revealed to other health-care personnel in order to provide appropriate health care. In this case, the nurse may be required to report information to the state health department. 4 Nurses should not make promises to keep necessary information private.

Which number of alternative solutions should be included when conducting ethical decision-making? 1) One 2) Two 3) Three 4) Four

ans 3 Feedback 1 One alternative solution is not the recommended number when implementing ethical decision-making. 2 Two alternative solutions are not the recommended number when implementing ethical decision-making. 3 The nurse should ensure that three alternative solutions are available when implementing ethical decision-making. 4 Four alternative solutions are not the recommended number when implementing ethical decision-making

The nurse is providing care to a patient who is diagnosed with multisystem fluid volume deficit. The patient is currently experiencing tachycardia and decreased urine output along with skin that is pale and cool to the touch. The patient has a decreased urine output. Which probable cause to the patient's symptoms should the nurse include when educating the family? 1) Congestive heart failure 2) Rapidly infused intravenous fluids 3) Natural compensatory mechanisms 4) Pharmacological effects of a diuretic

ans 3 Feedback 1 The manifestations reported are not indicative of cardiac failure in this client. 2 Rapidly infused intravenous fluids would not cause a decrease in urine output. 3 The internal vasoconstrictive compensatory reactions within the body are responsible for the symptoms exhibited. The body naturally attempts to conserve fluid internally specifically for the brain and heart. 4 A diuretic would cause further fluid loss, and is contraindicated

The nurse is providing care to an older adult patient who has decided to discontinue the prescribed hemodialysis. The patient's family, however, is not supportive of this decision. When using the theory of principles-based reasoning, which statement from the nurse is appropriate? 1) "The patient understands the decision and the advanced stage of the disease. If the patient quits treatment, the patient will die." 2) "I need to try to help the family understand the patient's decision so they can work through this situation together." 3) "This patient is of sound mind and is capable of making independent decisions regarding health care. It really is the patient's decision to make." 4) "This patient's health is so deteriorated that the treatment is not saving the patient's life. It is prolonging the ultimate outcome, which is death.

ans 3 Feedback 1 The patient's understanding of his decision and its consequences does not address the patient's right to make a decision autonomously. 2 Caring theories, or relationship theories, stress courage, generosity, commitment, and the need to nurture and maintain relationships. Caring theories promote the common good or the welfare of the group. Trying to help the family understand the patient's decision is an example of a caring-based theory in practice. 3 Principles-based theories stress individual rights, such as autonomy. The patient has the ability to make the decision, and it is his right to autonomy to do that. 4 Considering the patient's condition and the outcome of treatment is an example of consequence-based reasoning, in which the nurse looks at the outcomes of the patient's decision.

The nurse is providing care to patient with the following laboratory values: pH - 7.31; PaCO2 - 48 mmHg; and a normal HCO3. Which condition should the nurse plan care for based on the current data? 1) Metabolic acidosis 2) Metabolic alkalosis 3) Respiratory acidosis 4) Respiratory alkalosis

ans 3 Feedback 1 Uncompensated metabolic acidosis has a decreased pH, normal PaCO2, and normal HCO3. 2 Uncompensated metabolic alkalosis has an increased pH, normal PaCO2, and increased HCO3. 3 If the pH is decreased and the PaCO2 is increased with a normal HCO3, it is uncompensated respiratory acidosis. 4 Uncompensated respiratory alkalosis has an increased pH, decreased PaCO2, and normal HCO

The nurse is caring for a comatose patient with respiratory acidosis. For which intervention will the nurse need to collaborate when caring for this patient? 1) Monitoring vital signs 2) Measuring intake and output 3) Determining recent eating behaviors 4) Identifying current oxygen saturation level

ans 3 For patients in severe distress, family members may need to be consulted for critical information such as recent eating habits and history of vomiting.

The nurse is providing care to a patient who is admitted after a morphine overdose. Which acid-base imbalance should the nurse plan this patient's care to reflect? 1) Metabolic acidosis 2) Metabolic alkalosis 3) Respiratory acidosis 4) Respiratory alkalosis

ans 3 Morphine is a narcotic and generally acts to decrease or suppress respirations; therefore, this patient is probably hypoventilating. The expected acid-base imbalance would be respiratory acidosis

Which clinical manifestation supports the nurse's plan of care focusing on chronic respiratory acidosis? 1) Irritability 2) Blurred vision 3) Daytime sleepiness 4) Warm, flushed skin

ans 3 The manifestations of acute and chronic respiratory acidosis differ. The patient with chronic respiratory acidosis will demonstrate daytime sleepiness

A patient is admitted to the emergency department (ED) for fluid volume deficit. Which body system should the nurse focus to determine the cause of this imbalance when assessing this patient? 1) Genitourinary 2) Cardiovascular 3) Gastrointestinal 4) Musculoskeletal

ans 3 The most common cause of fluid volume deficit is excessive loss of gastrointestinal fluids, which can result from vomiting, diarrhea, suctioning, intestinal fistulas, or intestinal drainage. Other causes of fluid losses include chronic abuse of laxatives and/or enemas

A patient is admitted to the emergency department (ED) for fluid volume deficit. Which body system should the nurse focus to determine the cause of this imbalance when assessing this patient? 1) Genitourinary 2) Cardiovascular 3) Gastrointestinal 4) Musculoskeletal

ans 3 The most common cause of fluid volume deficit is excessive loss of gastrointestinal fluids, which can result from vomiting, diarrhea, suctioning, intestinal fistulas, or intestinal drainage. Other causes of fluid losses include chronic abuse of laxatives and/or enemas.

Which interprofessional role does the nurse often assume when providing patient care in an acute care setting? 1) Social worker 2) Client advocate 3) Care coordinator 4) Massage therapist

ans 3 The nurse often assumes the interprofessional role of care coordinator when providing patient care in an acute care setting.

The nurse is conducting a medication assessment for a preoperative patient. Which action by the nurse is appropriate for the patient who is prescribed phenobarbital? 1) Obtaining a baseline ECG 2) Monitoring blood pressure 3) Maintaining the drug during the perioperative period 4) Assessing blood glucose levels closely during the perioperative period

ans 3 The prescribed drug is a medication used to control seizures; therefore, this drug should be maintained during the perioperative period.

The nurse is reviewing the latest arterial blood gas results for a patient with metabolic alkalosis. Which result indicates that the metabolic alkalosis is compensated? 1) pH 7.32 2) HCO3 8 mEq/L 3) PaCO2 48 mmHg 4) PaCO2 18 mmHg

ans 3 To compensate for this imbalance, the rate and depth of respirations decrease, leading to retention of carbon dioxide. The PaCO2 will be elevated.

The nurse is analyzing the intake and output record for a patient being treated for dehydration. The patient weighs 176 lbs. and had a 24-hour intake of 2,000 mL and urine output of 1,200 mL. Based on this data, which conclusion by the nurse is the most appropriate? 1) Treatment has not been effective. 2) Treatment needs to include a diuretic. 3) Treatment is effective and should continue. 4) Treatment has been effective and should end.

ans 3 Urinary output is normally equivalent to the amount of fluids ingested; the usual range is 1,500-2,000 mL in 24 hours, or 40-80 mL in 1 hour (0.5 mL/kg per hour). Patients whose intake substantially exceeds output are at risk for fluid volume excess; however, the patient is dehydrated. The extra fluid intake is being used to improve body fluid balance. The patient's output is 40 mL/hr, which is within the normal range

The nurse is analyzing the intake and output record for a patient being treated for dehydration. The patient weighs 176 lbs. and had a 24-hour intake of 2,000 mL and urine output of 1,200 mL. Based on this data, which conclusion by the nurse is the most appropriate? 1) Treatment has not been effective. 2) Treatment needs to include a diuretic. 3) Treatment is effective and should continue. 4) Treatment has been effective and should end

ans 3 Urinary output is normally equivalent to the amount of fluids ingested; the usual range is 1,500-2,000 mL in 24 hours, or 40-80 mL in 1 hour (0.5 mL/kg per hour). Patients whose intake substantially exceeds output are at risk for fluid volume excess; however, the patient is dehydrated. The extra fluid intake is being used to improve body fluid balance. The patient's output is 40 mL/hr, which is within the normal range.

The nurse is reviewing new orders provided by the health-care provider for a critical care patient with metabolic acidosis. Which prescription should the nurse question? 1) Draw serum potassium levels every two hours. 2) Draw arterial blood gas samples every two hours. 3) Administer one ampule of sodium bicarbonate now. 4) Begin intravenous infusion of 0.9% normal saline.

ans 3 administering bicarbonate to correct acidosis increases the risk for hypernatremia, hyperosmolality, and fluid volume excess. This is the order that the nurse should question before providing

Which action should the nurse implement when providing patient care in order to support The Joint Commission's (TJC) National Patient Safety Goals (NPSG)? 1) Silencing a cardiorespiratory monitor 2) Identifying each patient using one source 3) Determining patient safety issues upon admission 4) Decreasing the amount of pain medication administered

ans 3 dentification of patient safety risks is a NPSG identified by the TJC. Determining patient safety issues upon admission supports this NPSG.

Which laboratory test should the nurse anticipate for a patient who reports chronic inflammation? 1) Varicella titer 2) Type and crossmatch 3) Erythrocyte sedimentation rate (ESR) 4) Complete blood count (CBC), with differentia

ans 3 in ESR screens for the presence of the inflammatory process.

Which did the Nursing Executive Center of The Advisory Board identify as an academic-practice gap for new graduate nurses? 1) Patient advocacy 2) Patient education3) Disease pathophysiology 4) Therapeutic communication

ans 3 knowledge of pathophysiology of patient conditions is identified as an academic practice gap for new graduate nurses

The nurse is preparing a patient, diagnosed with asthma, for surgery. Which should the nurse include in the plan of care for this patient? 1) Monitoring blood pressure every hour 2) Assessing bowel sounds twice per shift 3) Monitoring pulse oximetry continuously 4) Assessing deep tendon reflexes every hour

ans 3 patient diagnosed with asthma, who is scheduled for surgery, may have difficulty being weaned from the mechanical ventilator. This patient would require continuous pulse oximetry and arterial blood gas analysis in the plan of care.

The nurse is providing care to a patient who is diagnosed with multisystem fluid volume deficit. The patient is currently experiencing tachycardia and decreased urine output along with skin that is pale and cool to the touch. The patient has a decreased urine output. Which probable cause to the patient's symptoms should the nurse include when educating the family? 1) Congestive heart failure 2) Rapidly infused intravenous fluids 3) Natural compensatory mechanisms 4) Pharmacological effects of a diuretic

ans 3 the internal vasoconstrictive compensatory reactions within the body are responsible for the symptoms exhibited. The body naturally attempts to conserve fluid internally specifically for the brain and heart

The nurse is providing care to several patients on a medical-surgical unit. Which situation would necessitate the nurse to use SBAR during the hand-off process? 1) Wound care 2) Discharge to home 3) Transfer to radiology 4) Medication education

ans 3 the nurse should identify a patient using two sources prior to medication administration.

The nurse is conducting a medication assessment for a preoperative patient. Which action by the nurse is appropriate for the patient who is prescribed dexamethoasone? 1) Obtaining a baseline ECG 2) Monitoring blood pressure 3) Assessing for hyperglycemia 4) Tapering the drug two days prior to surgery

ans 3 the prescribed drug is a corticosteroid, therefore the moste appropriate nursing action is to assess the patient for hyperglycemia.

The case manager assembles a team of health-care professionals, including the patient's primary health-care provider, physical therapist, and social worker, for the purpose of collaborative discharge planning and decision making. Which type of team did the case manager assemble? Select all that apply. 1) Management 2) Intradisciplinary 3) Interdisciplinary 4) Interprofessional 5) Primary nursing car

ans 3,4 Feedback 1. This is incorrect. Management teams are executive-level teams that run the day-to-day operations of a corporation. 2. This is incorrect. Intradisciplinary teams include members of the same profession. 3. This is correct. Interdisciplinary teams include professionals of varied backgrounds who share decision making. The terms interprofessional team and interdisciplinary team are synonymous.4. This is correct. Interdisciplinary teams include professionals of varied backgrounds who share decision making. The terms interprofessional team and interdisciplinary team are synonymous. 5. This is incorrect. A primary nursing care team includes a primary nurse and associate nurses who will provide care to a patient during a hospital stay.

Which should the nurse be aware of when preparing to act as a patient advocate in the hospital setting? Select all that apply. 1) The rights of a patient in a long-term care facility 2) The health department's patient rights statement 3) The hospital's patient rights statement 4) State and federal patient rights legislation 5) The unit policy manual

ans 3,4 1. This is incorrect. The rights of a patient in a long-term care facility are not applicable when providing care in the hospital setting. 2. This is incorrect. The rights of a patient in the health department setting are not applicable when providing care in the hospital setting. 3. This is correct. The hospital's patient rights statement will assist the nurse to act as a patient advocate in the hospital setting. 4. This is correct. The state and federal patient rights legislation is applicable to patients in the hospital setting; therefore, the nurse should have knowledge of this information when acting as a patient advocate. 5. This is incorrect. The unit's policy manual will not have a separate policy statement from the hospital regarding the patient's rights.

26. The nurse is providing care to a patient who is exhibiting clinical manifestations of a fluid and electrolyte deficit. Based on this data, which health-care provider prescriptions does the nurse prepare to implement? Select all that apply. 1) Administer diuretics 2) Administer antibiotics 3) Initiate hypodermoclysis 4) Closely monitor patient's I&O's 5) Initiate intravenous therapy

ans 3,4,5, Feedback 1. This is incorrect. Diuretics may be ordered to reduce fluid volume excess. 2. This is incorrect. Antibiotics are not used for fluid and electrolyte imbalance. 3. This is correct. Hypodermoclysis, fluid administered subcutaneously, may be employed as a fluid delivery method, especially among older adults. 4. This is correct. Monitoring patient's intake and output is one of several ways to assess the patient's fluid status. 5. This is correct. Intravenous fluids may be ordered for the patient with a fluid volume deficit if replacement oral fluids cannot be taken in sufficient quantity

The nurse is providing care to a patient who is prescribed furosemide as part of the treatment for congestive heart failure (CHF). The patient's serum potassium level is 3.4 mEq/L. Which food should the nurse encourage the patient to eat based on this data? 1) Peas 2) Iced tea 3) Bananas 4) Baked fish

ans 3A potassium level of 3.4 is low, so the client should be encouraged to consume potassium-rich foods. Of the foods listed, the highest in potassium is banana.

16. The nurse is providing care to an adult patient admitted with dehydration and hyponatremia. Which medical condition supports the current nursing diagnosis of Electrolyte Imbalance? 1) Osmotic pressure 2) Hydrostatic pressure 3) Isotonic dehydration 4) Hypotonic dehydration

ans 4 Feedback 1 Osmotic pressure pulls fluid into the capillaries, usually in response to the presence of albumin and other plasma proteins made by the liver. 2 Hydrostatic pressure occurs when extracellular fluid volume excess occurs; the increased fluid volume in the vascular compartment congests the veins. 3 Isotonic dehydration occurs when fluid loss is not balanced by intake, and the losses of water and sodium are in proportion. 4 Hypotonic dehydration occurs when fluid loss is characterized by a proportionately greater loss of sodium than water, causing serum sodium to fall below normal levels

Which ethical principle requires the nurse to be accountable for commitments made to self or others? 1) Beneficence 2) Autonomy 3) Justice 4) Fidelity

ans 4 Feedback 1 Beneficence asks the question who benefits from the actions taken by others. 2 Autonomy examines an individual person's right to make decisions while providing acknowledgement and respect for the person's choices. 3 Justice examines who will be vulnerable to any actions taken. 4 Fidelity requires the nurse to be accountable for commitments made to others and self.

The nurse is providing care to an older adult patient who is scheduled for surgery. During the preoperative assessment, the nurse discovers that the patient does not have an adequate understanding of the procedure. Which is the reason for the nurse to take action in this situation? 1) The patient is very old and has multiple health problems. 2) The family needs to agree to the surgery. 3) The nurse witnessed the consent. 4) The patient has a right to informed consent.

ans 4 Feedback 1 The patient's age and health problems are not the reasons for the nurse to take action. 2 The family does not make the decision regarding surgery unless the patient has been declared incompetent by the court. 3 The nurse would want to have the surgery explained for the client's sake, not because the nurse signed the form. 4 The nurse should notify the surgeon because the patient has the right to informed consent.

The nurse is caring for a patient who is receiving intravenous fluids postoperatively following cardiac surgery. The nurse is aware that this patient is at risk for fluid volume excess. The family asks why the patient is at risk for this condition. Which response by the nurse is the most appropriate? 1) "Fluid volume excess is caused by inactivity." 2) "Fluid volume excess is caused by the intravenous fluids." 3) "Fluid volume excess is caused by new onset liver failure caused by the surgery." 4) "Fluid volume excess is common due to increased levels of antidiuretic hormone in response to the stress of surgery."

ans 4 Antidiuretic hormone (ADH) and aldosterone levels are commonly increased following the stress response before, during, and immediately after surgery. This increase leads to sodium and water retention. Adding more fluids intravenously can cause a fluid volume excess and stress upon the heart and circulatory system

The client is admitted to the emergency department (ED) with symptoms of a panic attack. Based on this data, the nurse plans care for which health problem? 1) Emesis 2) Memory loss 3) Hypoventilation 4) Respiratory alkalosis

ans 4 Anxiety disorders increase the risk for the acid-base imbalance respiratory alkalosis, due to hyperventilation that accompanies anxiety and panic attacks

Which nursing action exemplifies the Quality and Safety Education for Nursing (QSEN) competency of safety? 1) Advocating for a patient who is experiencing pain 2) Considering the patient's culture when planning care 3) Evaluating patient learning style prior to implementing discharge instructions 4) Assessing the right drug prior to administering a prescribed patient medication

ans 4 Assessing the right drug prior to administering a prescribed medication exemplifies the QSEN competency of safety.

the case manager interviews an older adult patient hospitalized after hip replacement surgery. The patient requires in-patient rehabilitation prior to being discharged home. The case manager works with the hospital nursing staff, the rehabilitation center, the patient's family members, and other care providers to assist with a smooth transition. Which is the primary goal of the care management model described here? 1) To provide greater peace of mind for the patient and his or her family members 2) To track a patient's progress to ensure that appropriate care is provided until discharge 3) To manage concerns that are related to the patient's medical care and treatment regimen only 4) To provide a continuum of clinical services in order to help contain costs and improve patient outcomes

ans 4 Case managers coordinate patient care to help ensure that a continuum of clinical services is provided. The goal of case management is to improve patient outcomes and to help contain costs

The medical-surgical unit recently implemented a patient-centered care model. Which action implemented by the nurse supports this model? 1) Evaluating care 2) Assessing needs 3) Diagnosing problems 4) Providing compassion

ans 4 Compassion is a competency closely associated with patient-centered care; therefore, this action supports the patient-centered model of care.

A nurse is working as the designated leader of a group of health-care providers in a community clinic setting. The team members are working to decrease the number of adolescent pregnancies in the community. They have defined the problem and are now focusing on objectives and considering various viewpoints presented by the group. The nurse is tasked with helping the team to stay focused in order to address the defined problem. Which competency of collaboration does this describe? 1) Trust 2) Mutual respect 3) Communication 4) Decision making

ans 4 Decision making involves shared responsibility for the outcome. The team must follow specific steps of the decision-making process, beginning with a clear definition of the problem. Team decision making must be directed at the objectives of the effort and requires full consideration and respect for various and diverse viewpoints, and often requires guidance and direction from a group leader

The nurse is providing care to a patient following hemodialysis. The patient is experiencing tachycardia and decreased urine output along with skin that is pale and cool to the touch. Which goal of hemodialysis does the nurse determine the patient has not met based on the current data? 1) Cardiac decompensation 2) A reduction of extracellular fluid 3) The effects of rapidly infused intravenous fluids 4) The pharmacological effects of a diuretic infused in the dialysate

ans 4 Diuretics and IV fluids are not administered during hemodialysis.

Which is the basis of nursing care practices and protocols? 1) Assessment 2) Evaluation 3) Diagnosis 4) Research

ans 4 Evidence that is obtained through research is the basis for nursing care practices and protocols.

A patient is admitted to the emergency department (ED) for dehydration. The patient is 154 lbs. Which urine output indicate the rehydration efforts for this patient have been effective? 1) 20 mL/hr 2) 25 mL/hr 3) 30 mL/hr 4) 35 mL/hr

ans 4 Expected urine output for an adult patient is 0.5 mL/kg/hr. The patient currently weighs 70 kg; therefore, adequate urine output would be at least 35 mL/hr.

Which professional value is the nurse demonstrating by volunteering time to work in a local free clinic? 1) Human dignity 2) Integrity 3) Altruism 4) Social justice

ans 4 Feedback 1 Human dignity is respect for the worth and uniqueness of individuals and populations. 2 Integrity is acting in accordance with an appropriate code of ethics and accepted standards of practice. 3 Altruism is concern for the welfare and well-being of others. 4 Social justice is upholding fairness on a social scale. This value is demonstrated in professional practice when the nurse works to ensure equal treatment under the law and equal access to quality health care.

A patient is admitted to the emergency department for the treatment of a drug overdose causing acute respiratory acidosis. Which substance noted on the toxicology report is the most likely cause for the current diagnosis? 1) PCP 2) Cocaine 3) Marijuana 4) Oxycodone

ans 4 Oxycodone is an opiate narcotic. Excessive use or overdose of narcotic substances can lead to respiratory depression and respiratory acidosis

A patient is prescribed 20 mEq of potassium chloride due to excessive vomiting. Which is the rationale for this drug the nurse should provide to the patient? 1) It controls and regulates water balance in the body. 2) It is used in the body to synthesize ingested protein. 3) It is vital in regulating muscle contraction and relaxation. 4) It is needed to maintain skeletal, cardiac, and neuromuscular activity.

ans 4 Potassium is the major cation in intracellular fluids, with only a small amount found in plasma and interstitial fluid. Potassium is a vital electrolyte for skeletal, cardiac, and smooth muscle activity.

The nurse is caring for a patient with a potassium level of 5.9 mEq/L. The health-care provider prescribes both glucose and insulin for the patient. The patient's spouse asks, "Why is insulin needed?" Which response by the nurse is the most appropriate? 1) "The insulin will help his kidneys excrete the extra potassium." 2) "The insulin is safer than other medications that can lower potassium levels." 3) "The insulin lowers his blood sugar levels and this is how the extra potassium is excreted." 4) "The insulin will cause his extra potassium to move into his cells, which will lower potassium in the blood."

ans 4 Serum potassium levels may be temporarily lowered by administering glucose and insulin, which cause potassium to leave the extracellular fluid and enter cells.

The nurse is providing care to a patient diagnosed with end-stage renal disease. When planning a care plan conference for this patient, who does the nurse invite to participate? 1) The oncologist 2) The psychiatrist 3) The hospital CEO 4) The family members

ans 4 The choice of health-care professionals who are invited to attend the conference is based on the needs of the patient. Family members are an important part of the care plan conference, especially for patients who are unable to advocate for themselves.

The patient is receiving sodium bicarbonate intravenously (IV) for correction of acidosis secondary to diabetic coma. The nurse assesses the patient to be lethargic, confused, and breathing rapidly. Which is the nurse's priority response to the current situation? 1) Stop the infusion and notify the provider because the patient is in alkalosis. 2) Increase the rate of the infusion and continue to assess the patient for symptoms of acidosis. 3) Decrease the rate of the infusion and continue to assess the patient for symptoms of alkalosis. 4) Continue the infusion, because the patient is still in acidosis, and notify the provider

ans 4 The client continues to exhibit signs of acidosis; symptoms of acidosis include lethargy, confusion, CNS depression leading to coma, and a deep, rapid respiration rate that indicates an attempt by the lungs to rid the body of excess acid, and the provider should be notified.

The nurse is caring for a patient with rheumatoid arthritis who expresses the desire to remain active as long as possible. In order for the patient to meet this goal, what should the nurse prepare to do? 1) Tell the patient there is no hope. 2) Ask the patient the reason for the decision. 3) Teach the patient nutrition and joint exercises. 4) Refer the patient to the appropriate professionals.

ans 4 The number of patients with chronic diseases with health-care needs is increasing rapidly, and nurses and primary health-care providers cannot meet all of these patients' needs. When a patient expresses the desire to live as normally as possible, the nurse should refer the patient to professionals who can help the patient meet that goal.

The nurse is planning care for the patient with acute renal failure. The nurse plans the patient's care based on the nursing diagnosis of Excess Fluid Volume. Which assessment data supports this nursing diagnosis? 1) Wheezing in the lungs 2) Generalized weakness 3) Bowel sounds positive in four quadrants 4) Pitting edema in the lower extremities

ans 4 The patient in acute renal failure will likely be edematous, as the kidneys are not producing urine.

The nurse is caring for a patient admitted with renal failure and metabolic acidosis. Which clinical manifestation would indicate to the nurse that planned interventions to relieve the metabolic acidosis have been effective? 1) Tachypnea 2) Palpitations 3) Increased deep tendon reflexes 4) Decreased depth of respirations

ans 4 The patient with metabolic acidosis will have an increased respiratory rate and depth. Signs that care has been effective would include a decrease in the rate and depth of respirations.

The nurse is conducting a medication assessment for a preoperative patient. Which action by the nurse is appropriate for the patient who is prescribed warfarin? 1) Obtaining a baseline ECG 2) Monitoring blood pressure 3) Assessing for hyperglycemia 4) Tapering the drug two days prior to surgery

ans 4 The prescribed drug is an anticoagulant; therefore, the most appropriate nursing action is to teach the patient to taper the drug for 48 hours prior to the surgical procedure.

Which is the priority nursing action when providing patient care during the preoperative phase of care? 1) Ensuring NPO status 2) Monitoring vital signs 3) Obtaining informed consent 4) Completing a preoperative checklist

ans 4 The priority nursing action during the preoperative period is to complete the preoperative checklist prior to the patient being transferred to the surgical suite.

The staff nurse is communicating with the change nurse about the change of status of the patient. The nurse would begin her communication with which statement if correctly using the SBAR format? 1) "The patient's heartrate is 110." 2) "I think this patient needs to be transferred to the critical care unit." 3) "The patient is a 68-year-old male patient admitted last night." 4) "The patient is complaining of chest pain."

ans 4 This statement is the "S" in the SBAR communication. This is the situation information.

Which data collected by the nurse during the assessment process places the older adult patient at risk for dehydration? 1) Poor skin turgor 2) Body mass index of 20.5 3) Blood pressure of 140/98 mmHg 4) Water intake of 2 glasses per day

ans 4 a poor intake of water could indicate a loss of the thirst response, which occurs as a normal age-related change. Since the patient only ingests two glasses of water each day, this could indicate a reduction in the normal thirst response.

A patient who is recovering from coronary bypass surgery is placed on a critical pathway for extended care. Which patient statement indicates appropriate understanding of the plan of care? 1) "I cannot alter the critical pathway plan." 2) "I must be able to meet goals that are set for me." 3) "My insurance plan can deny payment if I do not meet goals." 4) "The chosen critical pathway can be altered to meet my needs."

ans 4 care maps, or critical pathways, are flexible enough to be adjusted and tailored to the patient's needs and wishes.

A patient is admitted to the emergency department (ED) for dehydration. The patient is 154 lbs. Which urine output indicate the rehydration efforts for this patient have been effective? 1) 20 mL/hr 2) 25 mL/hr 3) 30 mL/hr 4) 35 mL/hr

ans 4 expected urine output for an adult patient is 0.5 mL/kg/hr. The patient currently weighs 70 kg; therefore, adequate urine output would be at least 35 mL/hr.

The patient's case manager, diabetes educator, and dietician meet to discuss the patient's needs in preparation for discharge to home. The patient's primary health-care provider arrives and states, "I will be making all decisions regarding the patient's discharge care." With the primary health-care provider's decision to lead the team, the dynamic has shifted between which two types of teams? 1) Intradisciplinary to interdisciplinary team 2) Multidisciplinary to intradisciplinary team 3) Interprofessional to interdisciplinary team 4) Interdisciplinary to multidisciplinary team

ans 4 interdisciplinary teams include professionals of varied backgrounds who share in decision making. Multidisciplinary teams include members of varied backgrounds, but treatment decisions are made by one member-usually the primary health-care provider.

The nurse is evaluating the level of evidence found during a recent review of the literature. Which evidence carries the lowest level of support for a practice change? 1) Level IV 2) Level V 3) Level VI 4) Level VII

ans 4 the lower the numeric value of the evidence the greater the support for a change in practice. Level VII evidence carries the lowest level of support for a practice change.

The nurse administers the preoperative medication to the patient one hour before elective surgery, and then discovers the preoperative consent is not signed. Which action by the nurse is the most appropriate? 1) Have the patient sign the consent quickly, before the medication begins taking effect. 2) Have a family member or medical power of attorney sign the consent. 3) Send the patient to the holding area without a signed consent. 4) Notify the health-care provider that surgery will need to be canceled.

ans 4 the nurse will notify the health-care provider, who will need to cancel surgery until the preoperative medication is excreted and no longer affecting the patient's ability to make informed decisions, at which time the consent can be signed.

The nurse is planning care for the patient with acute renal failure. The nurse plans the patient's care based on the nursing diagnosis of Excess Fluid Volume. Which assessment data supports this nursing diagnosis? 1) Wheezing in the lungs 2) Generalized weakness 3) Bowel sounds positive in four quadrants 4) Pitting edema in the lower extremities

ans 4 the patient in acute renal failure will likely be edematous, as the kidneys are not producing urine.

The nurse is discussing follow-up care with a patient who is being discharged. The patient and family cross their arms and state angrily that the team's suggestions are not acceptable. Which response by the nurse is appropriate? 1) "We only want what's best for you." 2) "We will leave you alone to discuss your options." 3) "Perhaps you did not understand the recommendations." 4) "Let's discuss other options that might work well for you and your family.

ans 4 the patient is the center of the team, and the goal is to facilitate healing. There are always other options to consider to reach that goal. The nurse would discuss other options with the patient, which will most likely increase cooperation by the patient, who will feel in control as the decision is made.

Which information should the nurse collect during the health history that is conducted during the preoperative period? 1) Caretaker after discharge2) Oral intake over the last day 3) Date of last sexual encounter 4) Previous response to anesthesia

ans 4 the patient's previous response to anesthesia should be determined at this time.

The nurse is conducting medication teaching for a patient who is prescribed an epi-pen. Which statements made by the patient indicates the need for additional instruction? 1) "I will carry an epi-pen with me at all times." 2) "I will check the expiration date on my epi-pen regularly." 3) "I should hold the epi-pen in place for 10 seconds after injection." 4) "I should use the epi-pen to inject the drug into my abdominal wall."

ans 4 the pen is placed against the thigh, not the abdomen, for injection. This statement indicates the need for additional instruction.

The nurse is conducting a medication assessment for a preoperative patient. Which action by the nurse is appropriate for the patient who is prescribed insulin? 1) Obtaining a baseline ECG 2) Monitoring blood pressure 3) Holding the drug during the perioperative period 4) Assessing blood glucose levels closely during the perioperative period

ans 4 the prescribed drug is administered to control the patient's blood glucose level; therefore, the nurse should monitor the patient's blood glucose level closely during the perioperative period.

The nurse is preparing to document care provided to the patient during the day shift. The nurse documents that the patient experienced an increased pain level while ambulating which required an extra dose of pain medication; took a shower; visited with family; and ate a small lunch. Which information is important to include during the oral end-of-shift reporting? Select all that apply. 1) The last antibiotics given 2) The patient's taking a shower 3) The patient's visit with family 4) The extra dose of pain medication 5) The patient's response to ambulation

ans 4,5 Feedback 1. This is incorrect. Antibiotics are reflected on the medication administration record (MAR). 2. This is incorrect. Taking a shower does not need to be reported, only documented. 3. This is incorrect. Visiting with the family need not be mentioned at change of shift but should be documented. 4. This is correct. The nurse would also report any as-needed medications given and when they were last given. 5. This is correct. In order to provide for the patient's safety, the nurse would pass on the patient's response to ambulation so that the oncoming staff can take fall precautions.

The nurse is conducting a medication assessment for a preoperative patient. Which action by the nurse is appropriate for the patient who is prescribed metoprolol? 1) Obtaining a baseline ECG 2) Monitoring blood pressure3) Assessing for hyperglycemia 4) Tapering the drug two days prior to surgery

ans 5 The prescribed drug is an antihypertensive; therefore, the most appropriate nursing action is to monitor the patient's blood pressure.

Which type of nursing is the root of all other nursing practice areas? 1) Pediatric nursing 2) Geriatric nursing 3) Medical-surgical nursing 4) Mental health-psychiatric nursing

ans3 medical-surgical nursing is the root of all nursing practice as care provided here can be implemented in all other areas of nursing practice.

The nurse is preparing a patient for discharge who will be requiring physical therapy (PT) to rehabilitate after a total knee replacement. After reading the health-care provider's order for PT, which would be the nurse's initial action? 1) Teach the family the exercises needed for the patient. 2) Call home health and schedule a therapist to visit the home for therapy. 3) Set up appointments according to the order with the hospital PT department. 4) Discuss the various types of settings for therapy and have the patient choose the venue.

ans3 the nurse best exhibits the characteristic that the patient has a right to self determination by presenting the methods available for PT and answering the patient's questions about each so the patient can make an informed decision.

The nurse is providing care to a patient who is exhibiting clinical manifestations of a fluid and electrolyte deficit. Based on this data, which health-care provider prescriptions does the nurse prepare to implement? Select all that apply. 1) Administer diuretics 2) Administer antibiotics 3) Initiate hypodermoclysis 4) Closely monitor patient's I&O's 5) Initiate intravenous therapy

ans3,4,5 Feedback 1. This is incorrect. Diuretics may be ordered to reduce fluid volume excess. 2. This is incorrect. Antibiotics are not used for fluid and electrolyte imbalance. 3. This is correct. Hypodermoclysis, fluid administered subcutaneously, may be employed as a fluid delivery method, especially among older adults. 4. This is correct. Monitoring patient's intake and output is one of several ways to assess the patient's fluid status. 5. This is correct. Intravenous fluids may be ordered for the patient with a fluid volume deficit if replacement oral fluids cannot be taken in sufficient quantity.

Which diagnostic test should the nurse anticipate when providing care to a patient diagnosed with chronic obstructive pulmonary disease (COPD) to monitor acid-base balance? 1) Pulse oximetry 2) Bronchoscopy 3) Sputum studies 4) Arterial blood gases

ans4 Feedback 1 Pulse oximetry is a noninvasive test that evaluates the oxygen saturation level of blood. 2 A bronchoscopy provides visualization of internal respiratory structures. 3 Sputum studies can provide specific information about bacterial organisms. 4 Arterial blood gas analysis is done to assess alterations in acid-base balance caused by respiratory disorders, metabolic disorders, or both.


Kaugnay na mga set ng pag-aaral

Unit 6 MCQThe construction of makeshift housing in a country within the periphery, as shown in the photograph, is often the result of

View Set

Counseling for Related Professions

View Set